Крок 3 - Медицина 2014 осінь (буклет)

1 / 200
Хворий 8-ми років протягом тижня скаржився на головний біль, втомлюваність. В анамнезі: бронхоаденіт у 4 роки. Стан погіршився, to- 37,8oC. Об’єктивно: дитина лежить на боці з приведеними до тулуба ногами, голова закинута. Ригідність м’язів потилиці, симптом Керніга. Ліквор прозорий, через 12 годин на поверхні утворилася плівка. Найбільш імовірний діагноз? An 8-year-old patient complained of headache and fatigue for a week. History: bronchoadenitis at the age of 4. The condition worsened, to- 37.8oC. About' objectively: the child lies on his side with his legs brought to the trunk, his head is thrown back. Stiffness of the muscles of the back of the head, Kernig's symptom. The cerebrospinal fluid is transparent, after 12 hours a film has formed on the surface. The most likely diagnosis?

Туберкульозний енцефаліт Tuberculous encephalitis

Туберкульозний менінгіт Tuberculous meningitis

Пневмококовий менінгіт Pneumococcal meningitis

Менінгококовий менінгіт Meningococcal meningitis

Стрептококовий менінгіт Streptococcal meningitis

2 / 200
Реанімаційна бригада швидкої медичної допомоги прибула на виклик до опіатного наркомана 23-х років. Об’єктивно: свідомість відсутня, ціаноз, брадипное, брадикардія. Що терміново показано хворому? The resuscitation team of emergency medical aid arrived on call to a 23-year-old opiate addict. Objectively: unconsciousness, cyanosis, bradypnea, bradycardia. What is urgently shown to the patient ?

Штучна вентиляція легень Artificial lung ventilation

Інфузійна терапія Infusion therapy

Призначення налорфіну Nalorphine Prescription

Форсований діурез Forced diuresis

Оксигенотерапія через маску Oxygenotherapy through a mask

3 / 200
На стаціонарному лікуванні перебуває вагітна жінка 33-х років. Строк вагітності 34 тижні. Впродовж останніх 2-х діб висуває скарги на головний біль, запаморочення, відчуття тяжкості в ділянці потилиці, зорові розлади. Мають місце набряки. АТ-160/120 мм рт.ст., ЧСС- 88/хв. В сечі: протягом доби зростаюча протеїнурія до 3,2 г/л. Діурез знижений. Яка найбільш імовірна патологія, яка обумовила цю симптоматику? A 33-year-old pregnant woman is undergoing inpatient treatment. She is 34 weeks pregnant. During the last 2 days, she complains of headache, dizziness, a feeling of heaviness in in the back of the head, visual disturbances. Swelling occurs. BP-160/120 mmHg, heart rate- 88/min. In the urine: increasing proteinuria up to 3.2 g/l during the day. Diuresis is reduced. What is the most likely pathology, what caused this symptomatology?

Гостре порушення мозкового кровообігу Acute cerebrovascular accident

Еклампсія Eclampsia

Гіпертонічний криз Hypertensive crisis

Тяжка прееклампсія Severe preeclampsia

Прееклампсія середньої тяжкості Preeclampsia of moderate severity

4 / 200
Хвора 22-х років після проходження курсу дезінтоксикаційної терапії вдома (в процесі якого був застосований один флакон розчину двічі на день впродовж 2-х діб) раптом відчула погіршення стану і була доставлена до приймального відділення. На момент огляду: відсутність свідомості, шкіра бліда, холодна, АТ- 50/0 мм рт.ст., Ps-132/хв. Яку невідкладну допомогу необхідно надати негайно? A 22-year-old patient, after completing a course of detoxification therapy at home (in the process of which one vial of the solution was used twice a day for 2 days), suddenly felt a worsening of her condition and was taken to the receiving department. At the time of examination: lack of consciousness, pale, cold skin, blood pressure - 50/0 mm Hg, Ps - 132/min. What emergency care should be provided immediately?

Корекція порушень розчином адреналіну Correction of violations with adrenaline solution

Лікування респіраторного дистрес-синдрому дорослих Treatment of respiratory distress syndrome in adults

Термінова інтубація трахеї Urgent tracheal intubation

Корекція показників кислотно-лужного стану Correction of indicators of acid-alkaline state

Негайне введення антибіотиків Immediate administration of antibiotics

5 / 200
Чоловік 46-ти років з імплантованим електрокардіостимулятором під час ремонту настільної лампи отримав легку травму електрострумом, після чого він впродовж декількох хвилин був непритомний. Об’єктивно: ціаноз шкіри, АТ- 60/0 мм рт.ст., ЧСС25 /хв. Найбільш імовірною причиною неефективності кровообігу є : A 46-year-old man with an implanted pacemaker received a slight electric shock during the repair of a table lamp, after which he was unconscious for several minutes. Objectively: cyanosis of the skin , blood pressure - 60/0 mm Hg, heart rate 25/min. The most likely cause of inefficiency of blood circulation is:

Ішемічний інсульт Ischemic stroke

Електромеханічна дисоціація Electromechanical dissociation

Тромбоемболія легеневої артерії Thromboembolism of the pulmonary artery

Атріовентрикулярна блокада Atrioventricular block

Травматичний шок Traumatic shock

6 / 200
Молодій жінці 5 днів тому в умовах стаціонару був здійснений аборт. В теперішній час висловлює скарги на слабкість, нудоту, остуду. Об’єктивно: притомна, психомоторне збудження з деякою неадекватністю поведінки. Шкірні покриви сухі, гарячі, температура 38,5oC, тахіпное 30/хв, у легенях дихання жорстке. АТ- 90/50 мм рт.ст., ЧСС- 112/хв. Діурез за останню добу знижений. Найбільш вірним буде припустити: A young woman had an abortion 5 days ago in a hospital. Now she complains of weakness, nausea, chills. Objectively: unconscious, psychomotor excitement with some inadequacy of behavior. The skin is dry, hot, temperature 38.5oC, tachypnea 30/min, breathing is hard in the lungs. Blood pressure - 90/50 mm Hg, heart rate - 112/min. Diuresis for the last day has decreased. The most faithful will assume:

Інфекційно-токсичний шок Infectious-toxic shock

Гострий пієлонефрит Acute pyelonephritis

Гостра пневмонія Acute pneumonia

Харчова токсикоінфекція Food poisoning

Гострий аднексит Acute adnexitis

7 / 200
Хлопчик 6-ти років скаржиться на загальну слабкість, генералізований свербіж та почервоніння шкіри, гостре підвищення температури тіла до 38oC. Із анамнезу: перший день хвороби. Протягом кількох годин після обробки невеликого садна іхтіоловою маззю з’явилися поширені гіперемовані папульозно-уртикарні висипання, що виступають над поверхнею шкіри в ділянці передпліччя, живота, стегон, гомілок. Укажіть найбільш імовірний діагноз: A 6-year-old boy complains of general weakness, generalized itching and reddening of the skin, a sharp increase in body temperature to 38oC. From the history: the first day of illness. Within a few hours after treatment of a small wound with ichthyol ointment, widespread hyperemic papular-urticarial rashes appeared, protruding above the surface of the skin in the area of ​​the forearm, abdomen, thighs, and lower legs. Specify the most likely diagnosis:

Набряк Квінке Quincke edema

Токсикодермія Toxicoderma

Алергічний контактний дерматит Allergic contact dermatitis

Генералізована кропивниця Generalized urticaria

Атопічний дерматит - нейродерміт, період загострення Atopic dermatitis - neurodermatitis, period of exacerbation

8 / 200
Хворий 13-ти років доставлений до відділення у коматозному стані. Відзначається глибоке шумне дихання за типом Кусмауля, гіпотонія, сухість шкіри і слизових оболонок, зниження тонусу очних яблук, олігурія, запах ацетону у видихуваному повітрі. Чим зумовлений даний стан? A 13-year-old patient was brought to the department in a comatose state. Deep, noisy Kussmaul-type breathing, hypotonia, dryness of the skin and mucous membranes, decreased tone of the eyeballs, oliguria, the smell of acetone in exhaled air. What is the cause of this condition?

Кетоацидотична кома Ketoacidotic Coma

Гіперосмолярна кома Hyperosmolar coma

Тиреотоксична кома Thyrotoxic Coma

Гіпоглікемічна кома Hypoglycemic coma

Печінкова кома Hepatic coma

9 / 200
Пацієнт 33-х років на роботі раптово відчув сильний головний біль, немов удар по голові. Знепритомнів, впав. Об’єктивно: АТ- 160/100 рт.ст., свідомість сплутана. Виражена ригідність м’язів шиї, симптом Керніга з обох боків під кутом 140°. Спостерігався генералізований судомний напад. Зіниці рівні, косоокості немає. Парезів немає. Яке обстеження необхідно провести хворому для виключення субарахноїдаль-ного крововиливу? A 33-year-old patient at work suddenly felt a severe headache, like a blow to the head. He fainted and fell. Objectively: blood pressure - 160/100 rt. st., consciousness is confused. Pronounced stiffness of the neck muscles, Kernig's sign on both sides at an angle of 140°. A generalized seizure was observed. The pupils are equal, there is no strabismus. There is no paresis. What examination should be performed on the patient to rule out subarachnoid hemorrhage?

Ангіографія Angiography

Електроенцефалограма Electroencephalogram

Електрокардіограма Electrocardiogram

Рентгенографія кісток черепа X-ray of skull bones

Люмбальна пункція Lumbar puncture

10 / 200
Дитина 6-ти років госпіталізована до дитячого відділення з бронхопневмонією. Страждає на атопічний дерматит. Після внутрішньом’язового введення ампіциліну з’явилися відчуття стиснення у грудях, запаморочення, різка блідість, ціаноз, холодний піт, прискорене шумне дихання. Який із перелічених препаратів слід увести в маніпуляційній негайно? A 6-year-old child was hospitalized in the children's ward with bronchopneumonia. He suffers from atopic dermatitis. After intramuscular administration of ampicillin, chest tightness, dizziness, sharp pallor, cyanosis, cold sweat, accelerated noisy breathing. Which of the listed drugs should be administered in the operating room immediately?

Розчин преднізолону внутрішньовенно Prednisone intravenous solution

Розчин еуфіліну внутрішньовенно Euphilin intravenous solution

Розчин тавегілу внутрішньовенно Tavegil intravenous solution

Розчин адреналіну підшкірно Solution of adrenaline subcutaneously

Розчин строфантину внутрішньовенно Strophanthine intravenous solution

11 / 200
У хворого болі в грудній клітці, які виникли після травми, задишка, слабкість. Об’єктивно: тахіпное, відставання правої половини грудної клітки в диханні, розширені вени шиї. Прогресує дихальна недостатність. Рентгенологічно: відсутність легеневого малюнка правої легені, середостіння зміщене вліво, на вдиху зміщується вправо. Купол діафрагми зміщений донизу. Який найбільш імовірний діагноз? The patient has chest pains that occurred after an injury, shortness of breath, weakness. Objectively: tachypnea, lagging of the right half of the chest in breathing, enlarged neck veins . Respiratory insufficiency is progressing. X-ray: absence of a pulmonary pattern of the right lung, the mediastinum is shifted to the left, on inhalation it shifts to the right. The dome of the diaphragm is shifted downward. What is the most likely diagnosis?

Емболія легеневої артерії Pulmonary embolism

Гемопневмоторакс Hemopneumothorax

Емфізема середостіння Emphysema of the mediastinum

Закритий пневмоторакс Closed pneumothorax

Напружений пневмоторакс Tense pneumothorax

12 / 200
Жінці 72-х років з артеріальною гіпертонією лікарем призначено новий препарат каптоприл у дозі 25 мг двічі на день. Вранці хвора вперше прийняла призначену дозу каптоприлу, відпочила, а коли встала - зненацька знепритомніла, впала. Об’єктивно: бліда, дихання поверхневе, часте, пульс слабкого наповнення, прискорений, артеріальний тиск низький. Яка найбільш імовірна патологія зумовила такий стан хворої? A 72-year-old woman with arterial hypertension was prescribed a new drug captopril at a dose of 25 mg twice a day. In the morning, the patient took the prescribed dose of captopril for the first time, rested, and when stood up - suddenly fainted, fell. Objectively: pale, breathing shallow, frequent, pulse weak, accelerated, blood pressure low. What is the most probable pathology that caused this state of the patient?

Інсульт Stroke

Гіпоглікемія Hypoglycemia

Ортостатичний колапс Orthostatic collapse

Гіперглікемія Hyperglycemia

Травма голови Head Injury

13 / 200
Хворий 40-ка років тиждень тому переніс 'на ногах' ГРВІ. Стан поступово погіршувався. Об’єктивно: млявий, сонливий, шкіра сірувато-блідого кольору, суха. В легенях ослаблене дихання, місцями сухі хрипи. ЧД- 26/хв. Тони серця глухі, аритмічні, ЧСС- 104/хв., АТ90 /65 мм рт.ст. Живіт м’який, печінка +3-4 см. Діурез 120 мл за добу. Який найбільш імовірний діагноз? A 40-year-old patient suffered an ARVI 'on his feet' a week ago. His condition gradually worsened. Objectively: lethargic, sleepy, grayish-pale skin, dry Weakened breathing in the lungs, sometimes dry wheezing. Heart rate - 26/min. Heart sounds are dull, arrhythmic, heart rate - 104/min., BP90 /65 mm Hg. Abdomen is soft, liver +3-4 cm. Diuresis 120 ml per day. What is the most likely diagnosis?

Тромбоемболія легеневої артерії Thromboembolism of the pulmonary artery

Бронхопневмонія Bronchopneumonia

Гостра ниркова недостатність Acute renal failure

Гостра серцево-судинна недостатність Acute cardiovascular failure

Гострий міокардит Acute myocarditis

14 / 200
Чоловіка 26-ти років вжалила бджола в ділянку рота. Через 15 хвилин виник набряк губ; язик значно збільшився у розмірах. Ще через 20 хвилин з’явився головний біль, температура тіла піднялася до 38°C. Об’єктивно: губи та язик значно набряклі. В ділянці повік спостерігається блідий інфільтрат, при натисканні на який не залишається ямки. Яка патологія найбільш імовірна у потерпілого? A 26-year-old man was stung by a bee in the mouth. After 15 minutes, swelling of the lips appeared; the tongue significantly increased in size. After another 20 minutes, a headache appeared , the body temperature rose to 38°C. Objectively: the lips and tongue are significantly swollen. In the area of ​​the eyelids, a pale infiltrate is observed, which does not leave a pit when pressed. What pathology is most likely in the victim?

Токсична реакція на отруту бджоли Toxic reaction to bee venom

Дерматоміозит Dermatomyositis

Бешиха Бешиха

Макрохейліт Мішера Mischer macrocheilite

Гострий набряк Квінке Acute Quincke's edema

15 / 200
Хвора 26-ти років збуджена, поведінка неадекватна, свідомість сплутана, розвинулося психомоторне збудження. В анамнезі - цукровий діабет І типу протягом 4-х років, добова доза інсуліну 54 ОД. Об’єктивно: шкіра волога, холодна, тургор нормальний, гіперрефлексія, зіниці розширені. АТ- 140/90 мм рт.ст., Ps- 88/хв. Глікемія 2,3 ммоль/л, аглюкозурія. Який найбільш імовірний діагноз? A 26-year-old patient is excited, her behavior is inadequate, her consciousness is confused, psychomotor agitation has developed. She has a history of type I diabetes for 4 years, a daily dose of insulin 54 units. Objectively: the skin is moist, cold, turgor is normal, hyperreflexia, pupils are dilated. Blood pressure - 140/90 mm Hg, Ps - 88/min. Glycemia 2.3 mmol/l, aglucosuria. Which is the most likely diagnosis?

Адисонічний криз Addisonian crisis

Вегетативний криз Vegetative crisis

Кетоацидотична кома Ketoacidotic Coma

Гіпоглікемічна кома Hypoglycemic coma

Тиреотоксична кома Thyrotoxic Coma

16 / 200
До лікарні бригадою ШМД доставлена жінка 32-х років. З анамнезу: протягом останніх 2х років знаходилась на диспансерному обліку з приводу аутоімунної гемолітичної анемії. Об’єктивно: стан середньої важкості, температура 37,3°C, шкіра лимонно-жовтого кольору, Ps- 98/хв. АТ-80/55 мм рт.ст., збільшена селезінка. У крові: ер.- 2,6 • 1012/л, Hb- 70 г/л, КП- 0,94, лейк.- 6,1 • 109/л, ШЗЕ- 19 мм/год, непрямий білірубін - 58 мкмоль/л. Що необхідно призначити в першу чергу? A 32-year-old woman was brought to the hospital by the medical team. From the anamnesis: for the past 2 years, she has been under medical records for autoimmune hemolytic anemia. Objectively: the condition of moderate severity, temperature 37.3°C, lemon-yellow skin, Ps- 98/min, BP-80/55 mm Hg, enlarged spleen. Blood: er.- 2.6 • 1012/l, Hb - 70 g/l, KP - 0.94, leuk. - 6.1 • 109/l, SZE - 19 mm/h, indirect bilirubin - 58 μmol/l. What should be prescribed in the first place?

Ферум-лек Ferum-lek

Трансфузія консервованої крові Transfusion of canned blood

Інтерферон Interferon

Преднізолон Prednisone

Еритроцитарна маса Erythrocyte mass

17 / 200
Чоловік 36-ти років скаржиться на інтенсивний біль у епігастральній ділянці, багаторазове блювання, яке не приносить полегшення, виражену загальну слабкість. Об’єктивно: загальний стан важкий, шкіра та видимі слизові - бліді, з поодинокими фіолетовими плямами. Ps- 100/хв., АТ-100/60 мм рт.ст. Під час пальпації живота визначається ригідність черевної стінки та різка болючість у епігастральній ділянці на 6 см вище пупка та у лівому реберно-хребетному куті. Пульсація аорти у епігастральній ділянці не визначається. Який найбільш імовірний діагноз? A 36-year-old man complains of intense pain in the epigastric area, repeated vomiting that does not bring relief, marked general weakness. Objectively: the general condition is severe, skin and visible mucous membranes - pale, with single purple spots. Ps- 100/min., BP-100/60 mm Hg. During palpation of the abdomen, rigidity of the abdominal wall and sharp pain in the epigastric area 6 cm above the navel and in in the left costo-spinal angle. Pulsation of the aorta in the epigastric area is not determined. What is the most likely diagnosis?

Розшаровуюча аневризма аорти Dissecting aortic aneurysm

Гострий панкреатит Hostry Pancreatitis

Гострий холецистит Hostry Cholecystitis

Гострий апендицит Acute appendicitis

Перфоративна виразка Perforative ulcer

18 / 200
До лікарні поступила дівчинка 14-ти років, яка 2 години тому із суїцидальними намірами випила 100 мл оцту. Дитина в свідомості, емоційно лабільна. Скарги на біль у роті та епігастрії, спрагу, слинотечу. При огляді - слизова оболонка рота гіперемована. Пальпаторно - напруження передньої черевної стінки. Для надання допомоги необхідно: A 14-year-old girl was admitted to the hospital, who 2 hours ago drank 100 ml of vinegar with suicidal intentions. The child is conscious, emotionally unstable. Complaints of pain in the mouth and epigastrium, thirst, salivation. On examination - the mucous membrane of the mouth is hyperemic. On palpation - tension of the anterior abdominal wall. To provide assistance, it is necessary:

Викликати штучне блювання Induce artificial vomiting

Не промивати шлунок, ввести активоване вугілля Do not wash the stomach, enter activated carbon

Промити шлунок розчином калію перманганату Wash the stomach with a solution of potassium permanganate

Промити шлунок холодною кип’яченою водою через зонд Rinse the stomach with cold boiled water through the tube

Промити шлунок розчином соди через зонд Wash the stomach with a soda solution through a tube

19 / 200
У хворого після амбулаторного лікування пневмонії виникли лихоманка, гіпертермія, озноб та кашель з виділенням харкотиння з неприємним запахом та прожилками крові. При аускультації легень справа в нижній частці амфоричне дихання, вологі хрипи. Рентгенологічно: справа в нижній частці порожнина до 4 см в діаметрі, з рівнем рідини. Яке ускладнення виникло у пацієнта? After outpatient treatment for pneumonia, the patient developed fever, hyperthermia, chills, and cough with sputum discharge with an unpleasant odor and streaks of blood. When auscultating the lungs in the right lower lobe, amphoric breathing , wet wheezing. X-ray: on the right, in the lower lobe, a cavity up to 4 cm in diameter, with a fluid level. What complication did the patient have?

Рак легень Lung cancer

Абсцес легень Lung abscess

Крупозна пневмонія Croup pneumonia

Інфільтративний туберкульоз Infiltrative tuberculosis

Гангрена легень Gangrene of the lungs

20 / 200
З перших годин життя у новонародженого відзначено задишку і ціаноз. Ліва половина грудної клітки відстає в акті дихання, міжреберні проміжки розширені. Живіт човноподібної форми, м’який. На рентгенограмі: множинні просвітлення у лівій плевральній порожнині у вигляді 'коміркової сітки'. Який діагноз найбільш імовірний? From the first hours of life, the newborn has shortness of breath and cyanosis. The left half of the chest lags behind in the act of breathing, the intercostal spaces are widened. The abdomen is boat-shaped, soft. On radiographs: multiple luminescence in the left pleural cavity in the form of a 'cellular grid'. What is the most likely diagnosis?

Спонтанний пневмоторакс Spontaneous pneumothorax

Вроджена діафрагмальна грижа Congenital diaphragmatic hernia

Вроджена напружена кіста легені Congenital tense lung cyst

Вроджена лобарна емфізема Congenital lobar emphysema

Ателектаз легені Atelectasis of the lung

21 / 200
Хвора 34-х років з приводу системного червоного вовчака отримує 30 мг преднізолону щоденно протягом 3-х років. Самостійно припинила прийом препарату. Під час огляду адинамічна. Шкіра бліда, з мармуровим відтінком, волога. Тони серця ритмічні, різко приглушені, ЧСС- 140/хв., пульс ниткоподібний. АТ- 80/20 мм рт.ст., температура тіла 38,8°C. Яку терапію слід призначити? A 34-year-old patient with systemic lupus erythematosus receives 30 mg of prednisone daily for 3 years. She stopped taking the drug on her own. During the examination, she is adynamic. The skin is pale , with a marble shade, wet. Heart sounds are rhythmic, sharply muffled, heart rate - 140/min., pulse is filamentous. BP - 80/20 mm Hg, body temperature 38.8°C. What therapy should be prescribed?

Цитостатики Cytostatics

Нестероїдні протизапальні препарати Nonsteroidal anti-inflammatory drugs

Преднізолон Prednisone

Симпатоміметики Sympathomimetics

Кардіотоніки Cardiotonics

22 / 200
Лікар надавав допомогу ураженим, що поступали з осередку хімічного зараження внаслідок аварії на хімічно небезпечному об’єкті. Індивідуальні засоби захисту не використовував. Незабаром погіршився зір, з’явились головний біль, запаморочення, задишка. Евакуйований в лікарню. Скаржиться на загальну слабкість, задишку. Об’єктивно: слизові ціанотичні, періодичні фібрилярні посіпування м’язів обличчя. Дихання утруднене, поверхневе. З боку серцево-судинної системи - виражена тахікардія, артеріальний тиск підвищений. Активність холінестерази крові пригнічена на 70%. Ураження отруйними речовинами (ОР) якої групи найімовірніше зумовлює таку клінічну картину? The doctor provided assistance to the injured who came from the center of chemical contamination as a result of an accident at a chemically dangerous facility. He did not use personal protective equipment. Soon his vision deteriorated, appeared headache, dizziness, shortness of breath. Evacuated to the hospital. Complains of general weakness, shortness of breath. Objectively: mucous cyanotic, periodic fibrillary twitching of facial muscles. Breathing is difficult, shallow. From the cardiovascular system - marked tachycardia, blood pressure increased. The activity of blood cholinesterase is suppressed by 70%. Injuries with poisonous substances (OR) of which group most likely cause such a clinical picture?

ОР шкірно-резорбтивної дії OR of skin resorptive action

ОР загальноотруйної дії Generally toxic OR

ОР подразливої дії OR of irritable action

ОР задушливої дії OR of suffocating action

ОР нервово-паралітичної дії OR of nerve-paralytic action

23 / 200
Хворий 42-х років скаржиться на головний біль, дратівливість, безсоння, зниження пам’яті і настрою. При неврологічному обстеженні виявлені слабопозитивні менінгеальні симптоми, анізокорія з млявою реакцією зіниць на світло. Ввечері раптово розвинувся стан потьмареної свідомості з зоровими галюцинаціями, страхом, психомоторним збудженням. При дослідженні ліквору виявлено велику кількість білку і клітинних елементів, позитивну реакцію Васермана. Який найбільш імовірний діагноз? A 42-year-old patient complains of headache, irritability, insomnia, decreased memory and mood. A neurological examination revealed weakly positive meningeal symptoms, anisocoria with a sluggish reaction pupils to the light. In the evening, a state of darkened consciousness with visual hallucinations, fear, psychomotor excitement suddenly developed. When examining the cerebrospinal fluid, a large amount of protein and cellular elements, a positive Wasserman reaction was found. What is the most likely diagnosis?

Алкогольний делірій Alcoholic delirium

Реактивний параноїд Reactive Paranoid

Деліріозна форма сифілісу мозку Delirious form of cerebral syphilis

Істеричний сутінковий стан Hysterical twilight state

Тривожно-депресивний розлад Anxiety-depressive disorder

24 / 200
У дівчинки 9-ти місяців з тетрадою Фалло виник напад збудження, задишки, ціанозу з епізодичною втратою свідомості. Об’єктивно: спостерiгається центральний ціаноз, над легенями - пуерильне дихання, перкуторно - розміри серця в межах норми. Аускультативно - грубий систолічний шум у другому міжребір’ї зліва від груднини, печінка виступає з-під ребра на 2 см. Яким має бути першочерговий крок лікаря поряд з проведенням оксигенотерапії? A 9-month-old girl with tetrad of Fallot had an attack of excitement, shortness of breath, cyanosis with episodic loss of consciousness. Objectively: central cyanosis is observed, puerile over the lungs breathing, percussion - the size of the heart is within the normal range. Auscultation - a rough systolic noise in the second intercostal space to the left of the sternum, the liver protrudes from under the rib by 2 cm. What should be the first step of the doctor along with oxygen therapy?

Призначення дигоксину Digoxin Prescription

Призначення морфіну Morpine Prescription

Призначення адреналіну Adrenaline Prescription

Призначення гідрокарбонату натрію Purpose of sodium bicarbonate

Призначення сальбутамолу Prescription of salbutamol

25 / 200
Чоловіка 40-ка років доставлено до приймального відділення. Відзначається запах алкоголю з рота, відкривання очей та мовні відповіді відсутні, нецілеспрямований рух на біль. Об’єктивно: дихання часте (>40/хв), поверхневе, АТ- 90/40 мм рт.ст., Ps-112/хв. Що треба зробити в першу чергу? A 40-year-old man was brought to the reception department. The smell of alcohol from the mouth is noted, there is no opening of the eyes and speech responses, no purposeful movement due to pain. Objectively: breathing frequent (>40/min), superficial, BP-90/40 mm Hg, Ps-112/min. What should be done first of all?

Форсований діурез Forced diuresis

Позаниркові методи детоксикації (гемодіаліз, гемосорбція) Extrarenal detoxification methods (hemodialysis, hemosorption)

Внутрішньовенно бемегрід Intravenous bemegrid

Інтубація трахеї Tracheal intubation

Зондове промивання шлунку Gastric tube lavage

26 / 200
У новонародженої дитини діагностовано клініку атрезії стравоходу. Назвіть найбільш вірне положення хворого з атрезією стравоходу з нижньою трахеостравохідною норицею під час транспортування: A newborn child was diagnosed with esophageal atresia. Name the most accurate position of a patient with esophageal atresia with a lower tracheoesophageal fistula during transportation:

Горизонтальне Horizontal

Положення не має значення The position does not matter

Вертикальне Vertical

На правому боці On the right side

З припіднятим тазовим кінцем With raised pelvic end

27 / 200
Чоловік 43-х років страждає на хронічний алкоголізм протягом 6-ти років. Останні 5 діб не приймає алкоголь. Скаржиться на головний біль, озноб, біль у ділянці серця. Тривожний, метушливий. Увечері посилився неспокій, рухове збудження, поривається втекти, сховатися. Змітає з ковдри 'комах, тарганів, павуків'. Оберіть оптимальну схему лікування: A 43-year-old man has been suffering from chronic alcoholism for 6 years. He has not taken alcohol for the last 5 days. He complains of headache, chills, pain in the heart area . Anxious, fussy. Restlessness increased in the evening, motor excitement, rushes to run away, hide. Sweeps 'insects, cockroaches, spiders' from the blanket. Choose the optimal treatment regimen:

Внутрішньовенно лазикс Intravenous Lasix

Внутрішньовенно галоперідол Intravenous haloperidol

Внутрішньовенно пірацетам Intravenous piracetam

Внутрішньовенно анальгін Intravenous analgin

Внутрішньовенно седуксен Intravenous seduxen

28 / 200
Вагітна 17-ти років у терміні 34-35 тижнів доставлена бригадою ШМД в пологовий будинок зі скаргами на головний біль, погіршення зору, посмикування м’язів, судоми. АТ190/100 мм рт.ст. на обох руках, набряки на ногах та животі. Стан плоду не порушений, виділень із статевих органів немає. Який найбільш імовірний діагноз? A 17-year-old pregnant woman at 34-35 weeks of age was taken to the maternity hospital by the medical team with complaints of headache, vision impairment, muscle twitching, convulsions. Blood pressure 190/100 mm Hg on both arms, swelling on the legs and abdomen. The condition of the fetus is not disturbed, there are no discharges from the genitals. What is the most likely diagnosis?

Прееклампсія тяжкого ступеня Severe preeclampsia

Еклампсія Eclampsia

Епілепсія Epilepsy

Гіпертонічна енцефалопатія Hypertensive encephalopathy

Менінгоенцефаліт Meningoencephalitis

29 / 200
До клініки дитячої хірургії доставлений новонароджений 3-х тижнів життя. На фоні повного благополуччя відзначається блювання 'фонтаном' впродовж останніх двох тижнів. Блювотні маси не містять жовчі. Запідозрений пілоростеноз. Який вид обстеження буде найінформативнішим? A 3-week-old newborn was brought to the children's surgery clinic. Against the background of complete well-being, there has been 'fountain' vomiting for the past two weeks. Vomiting masses do not contain bile. Suspect pylorostenosis. What type of examination will be the most informative?

Оглядова рентгенографія органів черевної порожнини Overview x-ray of abdominal organs

УЗД органів черевної порожнини Ultrasound of abdominal organs

Колоноскопія Colonoscopy

Лапароскопія Laparoscopy

Рентгенконтрастне дослідження шлунка X-ray contrast study of the stomach

30 / 200
Дитину 3-х років вжалила бджола у щоку. Розвинувся набряк язика, шиї, обличчя. Різка інспіраторна задишка з частотою 60/хв. Ps- 144/хв. АТ- 70/40 мм рт.ст. Аускультація легень: дихання різко ослаблене в нижніх відділах. Дитина різко збуджена. Який терапевтичний захід є першочерговим? A 3-year-old child was stung by a bee on the cheek. Swelling of the tongue, neck, face developed. Sharp inspiratory shortness of breath with a frequency of 60/min. Ps- 144/min. Blood pressure - 70/40 mm Hg. Auscultation of the lungs: breathing is sharply weakened in the lower parts. The child is sharply excited. What therapeutic measure is the first priority?

Лазикс внутрішньовенно Lasix IV

Обколоти постраждале місце адреналіном Surround the affected area with adrenaline

Інгаляція кисню Oxygen inhalation

Інтубація трахеї Tracheal intubation

Преднізолон внутрішньовенно Prednisone IV

31 / 200
Яка найбільш раціональна методика евакуації сечі з сечового міхура при гострій затримці сечовипускання внаслідок гострого паренхіматозного простатиту? What is the most rational method of evacuating urine from the bladder in acute urinary retention due to acute parenchymal prostatitis?

Епіцистостомія Epicystostomy

Катетеризація сечового міхура металевим катетером Catheterization of the urinary bladder with a metal catheter

Катетеризація сечового міхура еластичним катетером Catheterization of the urinary bladder with an elastic catheter

Надлонна капілярна пункція сечового міхура Suprapalm capillary puncture of the bladder

Троакарна цистостомія Trocar cystostomy

32 / 200
Хворий 40-ка років протягом тривалого часу хворіє на виразкову хворобу шлунка. Відмітив, що в останні 2 дні вираженність болю стала меншою, але з’явилися слабкість, запаморочення. Вранці, при вставанні з ліжка, він на кілька секунд знепритомнів. Об’єктивно: блідий. Пальпаторно в епігастральній ділянці незначна болісність. Симптоми подразнення очеревини відсутні. Які методи діагностики слід використати в першу чергу для верифікації даної патології? A 40-year-old patient has been suffering from peptic ulcer disease for a long time. He noticed that in the last 2 days the intensity of the pain has decreased, but weakness, dizziness have appeared . In the morning, when he got out of bed, he fainted for a few seconds. Objectively: pale. Palpation in the epigastric area slight pain. There are no symptoms of irritation of the peritoneum. What diagnostic methods should be used first of all to verify this pathology?

ФГДС ФГДС

Оглядова рентгенографія черевної порожнини Overview x-ray of the abdominal cavity

Рентгеноскопія шлунка Stomach X-ray

УЗД черевної порожнини Ultrasound of the abdominal cavity

Комп’ютерна томографiя черевної порожнини Computed tomography of the abdominal cavity

33 / 200
Пологи другі, вчасні. 3 години тому відійшли навколоплідні води. Пологова діяльність активна, загальнорівномірнозвужений таз II ступеня. Передбачувана маса плоду 3900 г. Серцебиття плоду не вислуховується. При піхвовому дослідженні відкриття шийки матки повне, плідного міхура немає, передлежить голівка плоду, притиснута до входу в малий таз. Стрілоподібний шов у поперечному розмірі. Яка лікарська тактика? The second birth, on time. 3 hours ago, the amniotic fluid left. Labor activity is active, generally uniformly narrowed pelvis of the II degree. Estimated weight of the fetus is 3900 g. The heartbeat of the fetus is not heard. vaginal examination, the opening of the cervix is ​​complete, there is no amniotic sac, the head of the fetus is presented, pressed against the entrance to the small pelvis. An arrow-shaped seam in the transverse dimension. What are the medical tactics?

Накладання акушерських щипців Applying obstetric forceps

Краніотомія Craniotomy

Стимуляція пологової діяльності окситоцином Stimulation of labor with oxytocin

Ведення пологів через природні статеві шляхи Giving birth through the natural genital tract

Кесарів розтин Caesarean section

34 / 200
На другу добу після субтотальної резекції щитоподібної залози з приводу дифузного токсичного зобу у хворої з’явилися тонічні судоми пальців ('рука акушера'), позитивні симптоми Хвостека, Вейса, Трусо. Яке ускладнення операції виникло? On the second day after subtotal resection of the thyroid gland due to diffuse toxic goiter, the patient developed tonic spasms of the fingers ('hand of the obstetrician'), positive symptoms of Khvostek, Weiss , Coward. What complication of the operation occurred?

Тиреотоксичний криз Thyrotoxic crisis

Асфіксія Asphyxia

Гіпопаратиреоз Hypoparathyroidism

Інфаркт міокарда Myocardial infarction

Анафілактичний шок Anaphylactic shock

35 / 200
Хворий поступив зі скаргами на напад серцебиття, ядуху, перебої в роботі серця. Подібні стани спостерігались і раніше. На ЕКГ: миготлива аритмія, ЧСС- 160/хв. ЕхоКГ: зниження фракції викиду до 45%. З якого препарату доцільно надавати невідкладну допомогу? The patient came in with complaints of palpitations, dyspnea, heart failure. Similar conditions were observed earlier. On the ECG: pulsatile arrhythmia, heart rate - 160/min. Echocardiogram: reduction of ejection fraction up to 45%. Which drug is appropriate to provide emergency care?

Новокаїнамід Novocaineamide

Верапаміл Verapamil

Кордарон Cordaron

Дигоксин Digoxin

Анаприлін Anaprilin

36 / 200
Хворий 38-ми років тривалий час хворіє на епілепсію. Вранці виникли часті великі судомні напади, в перервах між якими він залишався непритомним. На доторкання і больові подразники не реагує. Зіниці вузькі, реакція на світло квола. Тонус м’язів знижений. Наявні ознаки порушення дихання. З яких заходів необхідно розпочати надання невідкладної допомоги? A 38-year-old patient has been suffering from epilepsy for a long time. In the morning, there were frequent large convulsive attacks, in the intervals between which he remained unconscious. He does not respond to touch and painful stimuli Pupils are narrow, the reaction to light is weak. Muscle tone is reduced. There are signs of respiratory failure. What measures should be taken to provide emergency care?

Проведення дегідратаційної терапії Carrying out dehydration therapy

Проведення протисудомної терапії Conducting anticonvulsant therapy

Відновлення прохідності дихальних шляхів Restoration of airway patency

Попередження травматизації голови Head injury prevention

Проведення люмбальної пункції Lumbar puncture

37 / 200
Потерпілого 35-ти років доставлено до приймального відділення без свідомості. Брав участь у ліквідації аварії на хімічному виробництві, пов’язаному з галогенованими вуглеводнями. Об’єктивно: зіниці розширені, на світло не реагують, дихання Кусмауля, ригідність м’язів потилиці та кінцівок, позитивні рефлекси Бабінського, Гордона, Жуковського. Різко виражена жовтяниця, петехіальна висипка. Температура тіла - 37,6oC. Що зумовило розвиток вказаного стану в хворого? A 35-year-old victim was brought to the reception department unconscious. He participated in the liquidation of an accident at a chemical plant related to halogenated hydrocarbons. Objectively: pupils dilated, do not react to light, Kussmaul's breathing, stiffness of the muscles of the back of the head and limbs, positive reflexes of Babinsky, Gordon, Zhukovsky. Severe jaundice, petechial rash. Body temperature - 37.6oC. What caused the development of the indicated condition in the patient?

Гостра печінкова недостатність Acute liver failure

Гостра надниркова недостатність Acute adrenal insufficiency

Гостре порушення мозкового кровообігу Acute cerebrovascular accident

Геморагічна лихоманка Hemorrhagic fever

Гостра ниркова недостатність Acute renal failure

38 / 200
Хворий на перитоніт 40-ка років звернувся по медичну допомогу на 5 день після початку больового синдрому, блювання. Об’єктивно: тургор шкіри знижений, АТ-70/40 мм рт.ст. Ps- 124/хв., температура тіла 5oC. Діурез відсутній. З якого інфузійного розчину слід розпочати внутрішньовенну регідратацію? A 40-year-old patient with peritonitis sought medical help on the 5th day after the onset of pain syndrome, vomiting. Objectively: skin turgor decreased, blood pressure 70/ 40 mmHg. Ps- 124/min., body temperature 5oC. Diuresis is absent. With which infusion solution should intravenous rehydration be started?

0,9% натрію хлориду 0.9% sodium chloride

3% натрію хлориду 3% sodium chloride

4% натрію гідрокарбонату 4% sodium bicarbonate

1% калію хлориду 1% potassium chloride

10% глюкози 10% glucose

39 / 200
В лікарню доставлена жінка 22-х років зі скаргами на біль в нижніх відділах живота, рясні кров’янисті виділення зі статевих шляхів. Ps- 90/хв, АТ-100/70 мм рт.ст, живіт м’який. Остання менструація 6 тижнів тому. Який попередній діагноз? A 22-year-old woman was brought to the hospital with complaints of pain in the lower abdomen, profuse bloody discharge from the genital tract. Ps- 90/min, BP -100/70 mm Hg, the abdomen is soft. The last menstruation was 6 weeks ago. What is the previous diagnosis?

Апоплексія яєчника Ovarian apoplexy

Порушена позаматкова вагітність Disrupted ectopic pregnancy

Гострий апендицит Acute appendicitis

Самовільний викидень Spontaneous miscarriage

Тромбоз мезентеріальних судин Thrombosis of mesenteric vessels

40 / 200
Мати викликала лікаря до дитини 1 рік 7 місяців зі скаргами на раптовий кашель, задишку, які з’явились під час прийому їжі. При огляді: температура тіла у нормі, дитина активна. При аускультації легень: зліва свистячі хрипи на видиху, перкуторно коробковий звук. Який найбільш імовірний діагноз? The mother called the doctor to the child 1 year 7 months with complaints of a sudden cough, shortness of breath, which appeared during eating. On examination: the body temperature is normal , the child is active. On auscultation of the lungs: left whistling wheezing on exhalation, percussive box sound. What is the most likely diagnosis?

Бронхіоліт Bronchiolitis

Стороннє тіло бронху Foreign body of bronchus

Бронхіальна астма Bronchial asthma

Обструктивний бронхіт Obstructive bronchitis

Кашлюк Whooping cough

41 / 200
У хворого скарги на біль у правому підребер’ї, слабкість, блювання, пронос, які з’явилися 5 годин тому. В анамнезі 2 тижні назад тупа травма живота. Об’єктивно гіпотензія, тахікардія. Який найбільш імовірний діагноз? The patient complains of pain in the right hypochondrium, weakness, vomiting, diarrhea, which appeared 5 hours ago. He has a history of blunt abdominal trauma 2 weeks ago . Objectively hypotension, tachycardia. What is the most likely diagnosis?

Розрив селезінки Rupture of the spleen

Розрив кишечнику Intestinal rupture

Гострий апендицит Acute appendicitis

Перитоніт Peritonitis

Двоетапний розрив печінки Two-stage rupture of the liver

42 / 200
У постраждалого у ДТП перелом кісток таза. Відмічається уретрорагія. Нестерпні тенезми, які підсилюють біль. Виділення сечі немає. Пальпаторно і перкуторно визначається розтягнутий сечовий міхур. Перша медична допомога буде полягати в наступному: A victim of a traffic accident has a pelvic fracture. Urethrorrhea is noted. Unbearable tenesmus that increases the pain. There is no urine output. A distended bladder is determined by palpation and percussion. First medical aid will consist of the following:

Катетеризація сечового міхура по провіднику Catheterization of the urinary bladder through the conductor

Катетеризація сечового міхура металевим катетером Catheterization of the urinary bladder with a metal catheter

Катетеризація сечового міхура еластичним катетером Catheterization of the urinary bladder with an elastic catheter

Консервативне лікування (знеболювання, гемостатична терапія) Conservative treatment (analgesia, hemostatic therapy)

Надлобкова пункція сечового міхура Suprapubic bladder puncture

43 / 200
Хворий 45-ти років впав з мотоцикла. Скаржиться на біль у лівому стегні і гомілці. Об’єктивно: відмічається патологічна рухомість і крепітація кісткових уламків у нижній третині лівого стегна і верхній третині гомілки. Яку транспортну іммобілізацію слід застосувати для профілактики травматичного шоку? A 45-year-old patient fell off a motorcycle. He complains of pain in the left thigh and lower leg. Objectively: there is pathological mobility and crepitation of bone fragments in the lower third of the left thigh and the upper third of the lower leg. What type of transport immobilization should be used to prevent traumatic shock?

Прибинтувати одну кінцівку до іншої Bandage one limb to another

Шину Крамера Kramer's Tire

Шину Дітеріхса Tire of Dieterichs

Шину Крамера і прибинтувати хвору ногу до здорової Kramer's splint and bandage the sick leg to the healthy one

Дві шини Крамера Two Kramer Tires

44 / 200
Вантажник 36-ти років був притиснутий бортом автомобіля до стіни у ділянці грудної клітки. Скарги на загальну слабкість, задишку, біль і відчуття тяжкості в грудній клітці на боці ураження. Об’єктивно: блідість шкіри, тахікардія, пульс слабкого наповнення, артеріальний тиск знижений, симптоми недостатності дихання. Вкажіть обсяг первинної лікарської допомоги: A 36-year-old truck driver was pressed by the side of the car against the wall in the chest area. Complaints of general weakness, shortness of breath, pain and a feeling of heaviness in the chest on the affected side . Objectively: pallor of the skin, tachycardia, weak pulse, decreased blood pressure, symptoms of respiratory failure. Specify the amount of primary medical care:

Серцеві глікозиди, дихальні аналептики Cardiac glycosides, respiratory analeptics

Напівсидяче положення, анальгетики, інгаляція кисню Semi-sitting position, analgesics, oxygen inhalation

Негайна госпіталізація до травматологічного відділення Immediate hospitalization in the trauma department

Катетеризація центральної вени, інфузійна терапія Central venous catheterization, infusion therapy

Плевральна пункція у VII міжребір’ї Pleural puncture in the VII intercostal space

45 / 200
Хворий 47-ми років страждає на туберкульоз легенів 10 років. Періодично знаходиться на лікуванні в відділенні хронічних форм туберкульозу. Викликав 'швидку допомогу' у зв’язку з появою кровотечі. З ротової порожнини впродовж 1 години виділилось 0,5 літра пінистої яскраво-червоної крові. Куди треба госпіталізувати хворого? A 47-year-old patient has been suffering from pulmonary tuberculosis for 10 years. He is periodically being treated in the department of chronic forms of tuberculosis. He called an 'ambulance' in connection with the appearance bleeding. 0.5 liters of foamy, bright red blood was released from the oral cavity within 1 hour. Where should the patient be hospitalized?

Хірургічне відділення багатопрофільної лікарні Surgical department of a multidisciplinary hospital

Торакальне відділення протитуберкульозного диспансеру Thoracic department of anti-tuberculosis dispensary

Пульмонологічне відділення багатопрофільної лікарні Pulmonology department of a multidisciplinary hospital

Терапевтичне відділення протитуберкульозного диспансеру Therapeutic department of anti-tuberculosis dispensary

Терапевтичне відділення загальносоматичної лікарні Therapeutic department of the general somatic hospital

46 / 200
У дитини 8-ми років підвищення температури до 39°С, нежить зі значними гнійними виділеннями, вологий кашель, кон’юнктивіт з гнійним виділенням, світлобоязливість. На слизовій оболонці щік білуваті ділянки висівкоподібно злущеного епітелію. Який найбільш імовірний діагноз? An 8-year-old child has a temperature rise to 39°C, runny nose with significant purulent discharge, wet cough, conjunctivitis with purulent discharge, photophobia. On the mucous membrane the membranes of the cheeks are whitish areas of bran-like desquamated epithelium. What is the most likely diagnosis?

Кір Кір

Кореподібна краснуха Crush rubella

Висипний тиф Whispering Typhus

Грип Грип

Скарлатина Scarlet fever

47 / 200
Хвора звернулась зі скаргами на розширення вен правої нижньої кінцівки, наявність пігментації шкіри, набряку м’яких тканин правої гомілки. Права гомілка набрякла, шкіра гіперпігментована, багряно-синюшного кольору, гаряча на дотик. Яка подальша тактика? The patient complained of varicose veins of the right lower limb, the presence of skin pigmentation, swelling of the soft tissues of the right leg. The right leg is swollen, the skin is hyperpigmented, purple-bluish color, hot to the touch. What is the next tactic?

Туге бинтування кінцівки Tight bandaging of the limb

Призначення гепаринотерапії Prescription of heparin therapy

Консультація судинного хірурга Vascular surgeon consultation

Призначення венорутона Venoruton appointment

Консультація дерматолога Dermatologist consultation

48 / 200
У дитини 4-тижневого віку через 2 тижні від початку захворювання, що проявляється блюванням 'фонтаном', вираженою гіпотрофією і ексикозом, педіатр запідозрив вроджений пілоростеноз. При надходженні до дитячого відділення стан вкрай важкий: риси обличчя загострені, адинамія, сильна спрага, анурія, запах сечі з рота, сірість шкірних покривів. У сироватці крові: натрій - 135 моль/л, калій - 3,7 ммоль/л. Коматозний стан якого характеру розвинувся у дитини? In a 4-week-old child, 2 weeks after the onset of the disease, manifested by 'fountain' vomiting, pronounced hypotrophy and exicosis, the pediatrician suspected congenital pylorostenosis. Upon admission to in the children's department, the condition is extremely serious: facial features are aggravated, adynamia, strong thirst, anuria, the smell of urine from the mouth, gray skin. In the blood serum: sodium - 135 mol/l, potassium - 3.7 mmol/l. Comatose state of what nature developed in a child?

Кетоацидотична кома Ketoacidotic Coma

Азотемічна кома Azotemic coma

Гіперосмолярна кома Hyperosmolar coma

Гіпоглікемічна кома Hypoglycemic coma

Гіпохлоремічна кома Hypochloremic coma

49 / 200
У чоловіка 53-х років, хворого на цукровий діабет II типу, розвинулась кома. Глікемія - 36 ммоль/л, калій - 4,4 ммоль/л, натрій - 160 ммоль/л, рН крові - 7,24, стандартний бікарбонат - 20 ммоль/л, зсув буферних основ - 4 ммоль/л, сечовина - 16 ммоль/л. Який найбільш імовірний діагноз? A 53-year-old man with type II diabetes developed a coma. Glycemia - 36 mmol/l, potassium - 4.4 mmol/l, sodium - 160 mmol/l, blood pH - 7.24, standard bicarbonate - 20 mmol/l, buffer base shift - 4 mmol/l, urea - 16 mmol/l. What is the most likely diagnosis?

Гіперлактацидемічна кома Hyperlactacidemic coma

Гіперосмолярна кома Hyperosmolar coma

Гіпоглікемічна кома Hypoglycemic coma

Гостра серцева недостатність Acute heart failure

Діабетичний кетоацидоз Diabetic ketoacidosis

50 / 200
Хвора 66-ти років скаржиться на болі в животі, нудоту, блювання, болі в м’язах. Об’єктивно: різко виражені симптоми дегідратації, дихання Кусмауля, АТ- 90/50 мм рт.ст., анурія, температура 35,9oC, глікемія -12,9 ммоль/л, ацетонурія відсутня. рН крові 6,8, вміст молочної кислоти 1,7 ммоль/л (норма 0,62-1,3 ммоль/л). Який найбільш імовірний діагноз? A 66-year-old patient complains of abdominal pain, nausea, vomiting, muscle pain. Objectively: severe symptoms of dehydration, Kussmaul breathing, BP - 90/50 mmHg, anuria, temperature 35.9oC, glycemia -12.9 mmol/l, no acetonuria, blood pH 6.8, lactic acid content 1.7 mmol/l (norm 0.62 -1.3 mmol/l). What is the most likely diagnosis?

Гіперлактацидемічна кома Hyperlactacidemic coma

Мозкова кома Cerebral Coma

Гіперосмолярна кома Hyperosmolar coma

Уремічна кома Uremia Coma

Кетоацидотична кома Ketoacidotic Coma

51 / 200
20- річний чоловік хворий на інсуліно-залежний діабет, поступив зі скаргами на блювання, що тривало протягом 2-х днів, біль у животі та втрату свідомості. Газометричне дослідження крові вказувало на метаболічний ацидоз. Глюкоза крові 14,8 ммоль/л. Чим викликаний важкий стан пацієнта? A 20-year-old man with insulin-dependent diabetes came in with complaints of vomiting that lasted for 2 days, abdominal pain and loss of consciousness. Gasometric blood test indicated metabolic acidosis. Blood glucose 14.8 mmol/l. What is the cause of the patient's severe condition?

Діабетичний кетоацидоз Diabetic ketoacidosis

Інсульт Stroke

Гіпоглікемічна кома Hypoglycemic coma

Інфаркт міокарда Myocardial infarction

Хвороба Аддісона Addison's disease

52 / 200
У хлопчика 9-ти років скарги на загальну слабкість, підвищення температури до 37,8oC, висипання на шкірі. Об’єктивно: на шкірі еритема, набряк, множинні бульозні елементи. В анамнезі контакт з рослинами після дощу. Яке лікування необхідно призначити? A 9-year-old boy complains of general weakness, an increase in temperature up to 37.8oC, rash on the skin. Objectively: the skin has erythema, swelling, multiple bullous elements. History of contact with plants after rain. What treatment should be prescribed?

Антибіотики Antibiotics

Глюкокортикоїди Glucocorticoids

Ентеросорбенти Enterosorbents

Антигістамінні Antihistamines

Сечогінні Diuretics

53 / 200
У пологовий будинок доставлена вагітна без свідомості після трьох нападів еклампсії у терміні 32 тижні. Об’єктивно: стан дуже тяжкий, АТ- 180/120 мм рт.ст., анурія. Серцебиття плода глухе, 142/хв. Шийка матки сформована, канал шийки матки закритий. Тактика ведення вагітної? A pregnant woman was brought to the maternity hospital unconscious after three attacks of eclampsia at 32 weeks. Objectively: the condition is very serious, blood pressure - 180/120 mm Hg ., anuria. The fetal heartbeat is deaf, 142/min. The cervix is ​​formed, the cervical canal is closed. Tactics of managing a pregnant woman?

Передопераційна підготовка з ШВЛ протягом декількох годин, потім оперативне розродження Preoperative preparation with ventilator for several hours, then operative delivery

Інтенсивна терапія протягом декількох годин, потім кесарів розтин Intensive therapy for several hours, then cesarean section

Розродження шляхом кесаревого розтину в екстреному порядку Emergency cesarean delivery

Створення глюкозо-вітаміно-гормонально-кальцієвого фону у сукупності з комплексною інтенсивною терапією Creation of a glucose-vitamin-hormonal-calcium background in combination with complex intensive therapy

Комплексна терапія гестозу протягом доби, потім розродження Complex therapy of gestosis during the day, then delivery

54 / 200
Хвора 25-ти років перенесла тяжку форму фолікулярної ангіни. Поступово болі під час ковтання зменшились, температура нормалізувалась. Відчула себе здоровою. Але через 2 дні знову піднялася температура до 38oC, з’явились болі під час ковтання. Ліва частка щитоподібної залози збільшилась, стала щільною і різко болісною, шкіра над нею гіперемована. З’явились тахікардія, подразливість, пітливість. У крові: лейк.-14,0 • 109/л, ШЗЕ- 30 мм/год. Основний обмін +25%. Який найбільш імовірний діагноз? A 25-year-old patient suffered a severe form of follicular angina. Gradually, the pains during swallowing decreased, the temperature normalized. She felt healthy. But after 2 days, the temperature rose again to 38oC, pains appeared during swallowing. The left lobe of the thyroid gland increased, became dense and sharply painful, the skin over it was hyperemic. Tachycardia, irritability, sweating appeared. In the blood: leuk.-14.0 • 109/l, SZE- 30 mm/h. Basic exchange +25%. What is the most likely diagnosis?

Дифузний токсичний зоб Diffuse toxic goiter

Гострий тиреоїдит Acute thyroiditis

Паратиреоз Parathyroidism

Мікседема Myxedema

Рак щитоподібної залози Thyroid cancer

55 / 200
У дитини 10 місяців на тлі легкого перебігу ГРВІ з’явилися повторні клонічні судоми. При огляді виявлено затримку психомоторного розвитку, краніотабес, деформацію грудної клітки, потовщення метафізів трубчатих кісток. Рівень кальцію крові - 1,6 ммоль/л, інтервал QT на ЕКГ подовжений - 0,33 сек. Даних про перинатальне ушкодження ЦНС немає. Спинномозкова рідина інтактна, витікала під тиском. Вигодування штучне, без овочевих страв. Про яке захворювання йдеться? A 10-month-old child developed repeated clonic convulsions against the background of a mild acute respiratory viral infection. Examination revealed delayed psychomotor development, craniotabes, chest deformation, thickening of the metaphyses of the tubular bones Blood calcium level - 1.6 mmol/l, QT interval on ECG prolonged - 0.33 sec. There are no data on perinatal damage to the central nervous system. Cerebrospinal fluid is intact, it leaked out under pressure. Feeding is artificial, without vegetable dishes. What disease is it about?

Нейротоксикоз Neurotoxicosis

Енцефаліт Encephalitis

Енцефалітична реакція Encephalitic reaction

Менінгіт Meningitis

Спазмофілія Spasmophilia

56 / 200
У дитини 10-ти місяців, що постійно вигодовувалась коров’ячим молоком, відмічено виражений систолічний шум на верхівці серця та акроціаноз. Сонографічно вади серця не виявлено. У крові: Hb- 38 г/л, КП- 0,7. Першочергова терапія: A 10-month-old child who was constantly fed cow's milk had a pronounced systolic murmur at the top of the heart and acrocyanosis. Sonographically, no heart defects were detected. In the blood : Hb- 38 g/l, KP- 0.7. Primary therapy:

Серцеві глікозиди по швидкій схемі насичення Cardiac glycosides according to the fast saturation scheme

Трансфузія еритроцитарної маси Transfusion of erythrocyte mass

Введення рекормону Recormon input

Ентеральне введення препаратів заліза Enteral administration of iron preparations

Трансфузія цільної крові Whole blood transfusion

57 / 200
У недоношеної новонародженої дитини, що народилася від матері з ознаками ендометриту, сірий відтінок шкіри, млявість, гіперестезія, менінгеальні знаки, блювання, гіпотермія. Запідозрений гнійний менінгіт. Яке дослідження допоможе підтвердити діагноз? A premature newborn born to a mother with signs of endometritis has gray skin, lethargy, hyperesthesia, meningeal signs, vomiting, hypothermia. Suspected purulent meningitis. Which will the study help to confirm the diagnosis?

Дослідження сечі Urine study

Дослідження крові Blood test

Дослідження ліквору Liquor Research

Дослідження калу Stool study

Дослідження слини Saliva study

58 / 200
До сімейного лікаря звернулася дівчинка 8-ми років зі скаргами на підвищену пітливість, серцебиття, схуднення. З анамнезу: хвороба розвинулася 1,5 роки тому, коли з’явилися серцебиття і поступова втрата ваги. Лікування не отримувала. Об’єктивно: шкіра волога, Ps- 130/хв. АТ- 135/60 мм рт.ст. Щитоподібна залоза III ст., рухома, безболісна. Який попередній діагноз? An 8-year-old girl came to the family doctor with complaints of increased sweating, palpitations, weight loss. From the anamnesis: the disease developed 1.5 years ago, when palpitations and gradual weight loss appeared. She did not receive treatment. Objectively: moist skin, Ps- 130/min. BP- 135/60 mm Hg. Thyroid gland III stage, mobile, painless. What is the previous diagnosis?

Надмірні фізичні навантаження Excessive physical activity

Вроджена вада серця Congenital heart defect

Гігантизм Gigantism

Дифузний токсичний зоб Diffuse toxic goiter

Синдром мальабсорбції Malabsorption syndrome

59 / 200
У пацієнтки 16-ти років при введенні в/в ферум-леку виник сильний головний біль, шум у вухах, болі за грудниною, відчуття жару, пітливість. Об’єктивно: стан хворої важкий, Ps115 /хв., ниткоподібний, АТ-80/30 мм рт.ст. Який з препаратів необхідно ввести в першу чергу? A 16-year-old female patient developed a severe headache, tinnitus, pain behind the sternum, feeling hot, and sweating when the IV ferrum drug was administered. objectively: the patient's condition is severe, Ps115/min., filamentous, BP-80/30 mm Hg. Which of the drugs should be administered first?

Атропін Atropine

Адреналін Adrenaline

Мезатон Mezaton

Допамін Dopamine

Преднізолон Prednisone

60 / 200
Пацієнт 54-х років, що знаходиться в стаціонарі з приводу гострого інфаркту міокарда, раптово знепритомнів. При огляді діагностовані зупинка серцевої діяльності і дихання, розпочато легенево-серцеву реанімацію. На ЕКГ асистолія. Далі необхідно: A 54-year-old patient, who is in the hospital due to an acute myocardial infarction, suddenly fainted. Cardiac and respiratory arrest were diagnosed during the examination, cardiopulmonary resuscitation was started . Asystole on the ECG. Then it is necessary:

Ввести внутрішньосерцево адреналін Inject adrenaline intracardiacally

Дефібриляція Defibrillation

Ввести внутрішньовенно лідокаїн Inject lidocaine intravenously

Ввести внутрішньовенно новокаїнамід Enter intravenous novocaine

Ввести внутрішньовенно строфантин Enter strophanthin intravenously

61 / 200
Дитині 4 місяці. Мати скаржиться на блювання після нових видів їжі протягом декількох годин. Періодично у дитини з’являються напади хвилювання: різкий крик з підведенням ніг до черева. Об’єктивно: шкіра бліда, спостерігається тахікардія, живіт здутий, гази не відходять. Замість випорожнень - кров. Який найбільш імовірний діагноз? The child is 4 months old. The mother complains of vomiting after new types of food for several hours. Periodically, the child has fits of excitement: a sharp cry with legs brought to the stomach. Objectively: the skin is pale, tachycardia is observed, the abdomen is distended, gas does not pass. Instead of stool, there is blood. What is the most likely diagnosis?

Інвагінація Intussusception

Поліп прямої кишки Rectal polyp

Ентерит Enteritis

Виразкова хвороба Ulcer disease

Коліт Colitis

62 / 200
Хвора 18-ти років страждає на цукровий діабет I типу з 10-ти років. Отримує 60 ОД інсуліну на добу. Вранці поснідала недостатньо, через 2 години знепритомніла. Об’єктивно: свідомість відсутня. Дихання рівне, ритмічне. Шкіра волога, тонус м’язів кінцівок підвищений. АТ- 110/70 мм рт.ст. Ps- 80/хв. Прикушування язика немає. Менінгіальні симптоми відсутні. Що повинен виконати лікар швидкої допомоги? An 18-year-old patient has been suffering from type I diabetes since the age of 10. She receives 60 units of insulin per day. She did not have enough breakfast in the morning, she fainted after 2 hours. Objectively: there is no consciousness. Breathing is even, rhythmic. The skin is moist, the tone of the muscles of the limbs is increased. BP- 110/70 mm Hg. Ps- 80/min. There is no tongue biting. There are no meningeal symptoms. What should the doctor do ambulance?

Ввести 20 ОД інсуліну п/ш та транспортувати до лікарні Inject 20 units of insulin per day and transport to hospital

Налагодити в/в краплинне введення 5% р-ну глюкози та транспортувати до лікарні Set up intravenous drip of 5% glucose solution and transport to hospital

Налагодити в/в краплинне введення 0,9% р-ну натрію хлориду та транспортувати до лікарні Set up an intravenous drip of 0.9% sodium chloride and transport to the hospital

Доставити хвору у лікарню, де будуть виконані лікувальні заходи Deliver the patient to the hospital, where medical measures will be performed

Ввести хворій 40% р-н глюкози в/в струминно та транспортувати до лікарні Inject the patient with 40% glucose intravenously and transport to the hospital

63 / 200
Дитина 13-ти років, яка впродовж 3-х років хворіє на цукровий діабет, доставлена у відділення інтенсивної терапії. Після перенесеного грипу протягом 2-х тижнів з’явилися спрага, поліурія, втрата ваги. Поступово збільшувалася слабкість, з’явилася сонливість. Під час госпіталізації цукор крові - 20 ммоль/л, цукор сечі - 4%, кетонові тіла в сечі ++++. Діагностовано діабетичну (гіперглікемічну кетоацидотичну) прекому. Розпочата регідратаційна терапія ізотонічним розчином хлориду натрію. Яка початкова доза інсуліну має бути призначена? A 13-year-old child, who has been suffering from diabetes for 3 years, was taken to the intensive care unit. After having the flu for 2 weeks with thirst, polyuria, weight loss appeared. Weakness gradually increased, drowsiness appeared. During hospitalization, blood sugar - 20 mmol/l, urine sugar - 4%, ketone bodies in the urine ++++. Diabetic (hyperglycemic ketoacidotic) precoma was diagnosed . Rehydration therapy with an isotonic sodium chloride solution has been started. What initial dose of insulin should be prescribed?

0,05-0,1 ОД/кг/годину 0.05-0.1 OD/kg/year

1 ОД/кг/годину 1 OD/kg/year

0,5-1,0 ОД/годину 0.5-1.0 OD/year

0,5 ОД/кг/годину 0.5 OD/kg/year

1-1,5 ОД/кг/добу 1-1.5 OD/kg/dobu

64 / 200
Хвора 36-ти років звернулася до лікаря зі скаргами на біль ниючого характеру, який виник гостро в клубовій ділянці і попереку з іррадіацією в пахову ділянку справа. Біль виник після переохолодження. Під час пальпації болісність у клубовій ділянці справа. Підвищення температури тіла до 37,8°C. У крові: ШЗЕ- 22 мм/год, лейк.- 9,3 • 109/л. Який найбільш імовірний діагноз? A 36-year-old patient turned to the doctor with complaints of pain of an aching nature, which arose acutely in the iliac region and lower back with radiation to the inguinal region on the right. The pain occurred after hypothermia. During palpation, pain in the iliac region on the right. An increase in body temperature to 37.8°C. In the blood: SZE - 22 mm/h, leuk. - 9.3 • 109/l. What is the most likely diagnosis?

Загострення хронічного аднекситу Exacerbation of chronic adnexitis

Правобічний деформуючий коксартроз Right-sided deforming coxarthrosis

Злоякісна пухлина товстої кишки Malignant colon tumor

Вагоінсулярна криза Vagoinsular crisis

Люмбаго Lumbago

65 / 200
Хворий 30-ти років слюсар контрольно- вимірювальних приладів, звернувся до лікаря зі скаргами на 6іль у животі, послаблення випорожнень, різку слабкість, набухання та біль ясен, відчуття металевого присмаку у роті. Об’єктивно: набухання ясен, наявність виразок на них. Живіт болючий під час пальпації тонкого та товстого кишечнику, кров у калі. Пожвавленість сухожильних рефлексів. Яке захворювання найбільш імовірне у хворого? A sick 30-year-old fitter of control and measuring devices, turned to the doctor with complaints of abdominal pain, loose stools, sharp weakness, swelling and pain in the gums, feeling a metallic taste in the mouth. Objectively: swelling of the gums, the presence of ulcers on them. The abdomen is painful during palpation of the small and large intestines, blood in the stool. Invigoration of tendon reflexes. What disease is most likely in the patient?

СНІД AIDS

Гостре отруєння металевою ртуттю Acute poisoning with metallic mercury

Виразковий коліт Ulcerative colitis

Хвороба Крона Crohn's disease

Хронічне отруєння свинцем Chronic lead poisoning

66 / 200
Дитині 6-ти років, оперованій з приводу розлитого перитоніту, у процесі забезпечення анестезіологічного засобу, проводиться штучна вентиляція легень у режимі помірної гіпервентиляції. На яких цифрах необхідно підтримувати рівень pC02 у КЛС-грамі? A 6-year-old child operated on for spilled peritonitis, in the process of providing an anesthetic agent, artificial lung ventilation is performed in the mode of moderate hyperventilation. At what numbers should the level be maintained pC02 in KLS-gram?

20-25 мм рт.ст. 20-25 mmHg

35-40 мм рт.ст. 35-40 mmHg

40-45 мм рт.ст. 40-45 mmHg

50-55 мм рт.ст. 50-55 mmHg

30-35 мм рт.ст. 30-35 mmHg

67 / 200
Хвора 28-ми років доставлена до лікарні машиною швидкої допомоги без свідомості. Об’єктивно: запах ацетону у повітрі, що видихається, шкіра суха, язик обкладений коричневим нальотом, сухий, тонус очних яблук знижений, дихання за типом Кусмауля. Цукор крові - 25,2 ммоль/л, ацетон сечі ++, осмолярність крові - 280 мосмоль/л. Які невідкладні заходи потрібно вжити? A 28-year-old patient was brought to the hospital by ambulance unconscious. Objectively: the smell of acetone in the exhaled air, dry skin, a brown coating on the tongue , dry, the tone of the eyeballs is reduced, Kussmaul's breathing. Blood sugar - 25.2 mmol/l, urine acetone ++, blood osmolality - 280 mosmol/l. What urgent measures should be taken?

Внутрішньовенне струминне введення 40% розчину глюкози Intravenous jet injection of 40% glucose solution

Внутрішньовенне краплинне введення 5% розчину глюкози Intravenous drip of 5% glucose solution

Внутрішньовенне краплинне введення 2,5% розчину бікарбонату натрію Intravenous drip of 2.5% sodium bicarbonate solution

Внутрішньом’язове введення 1% розчину глюкагону Intramuscular injection of 1% glucagon solution

Проведення регідратації, дробне введення інсуліну Carrying out rehydration, fractional insulin injection

68 / 200
Хвора 54-х років, що страждає на гіпотиреоз, скаржилась на підвищену втомлюваність, сонливість, загальне нездужання, погану пам’ять, неможливість виконувати нескладну роботу, задишку, запори, значне збільшення ваги. Впродовж декількох місяців не приймала гормональну терапію. Стан поступово погіршувався. Дільничний лікар знайшов хвору у непритомному стані, дихання рідке, поверхневе, тони серця глухі, ЧСС- 45/хв., шкіра суха, холодна, живіт м’який. Які заходи потрібно було вжити раніше для недопущення розвитку коматозного стану? A 54-year-old patient suffering from hypothyroidism complained of increased fatigue, drowsiness, general malaise, poor memory, inability to perform simple work, shortness of breath, constipation, significant weight gain. For several months, she did not take hormonal therapy. The condition gradually worsened. The district doctor found the patient in an unconscious state, breathing was shallow, shallow, heart sounds were dull, heart rate - 45/min., skin was dry, cold, stomach was m which. What measures should have been taken earlier to prevent the development of a comatose state?

Призначити серцеві глікозиди Prescribe cardiac glycosides

Призначити сечогінні Prescribe diuretics

Призначити інсулін Prescribe insulin

Призначити замісну терапію тиреоїдни-ми гормонами Prescribe thyroid hormone replacement therapy

Призначити 5% розчин глюкози Prescribe 5% glucose solution

69 / 200
У спекотну погоду юнак 14-ти років відпочивав на пляжі. Через 6 годин вдома поскаржився на загальну слабкість, головний біль, запаморочення, нудоту й блювання. Об’єктивно: гіперемія шкіри обличчя та голови, одутлість обличчя. Шкірні покриви вологі. ЧД- 19/хв., АТ- 125/80 мм рт.ст., ЧСС=Р8=104/хв. Який найбільш імовірний діагноз? In hot weather, a 14-year-old boy rested on the beach. After 6 hours at home, he complained of general weakness, headache, dizziness, nausea and vomiting. Objectively : hyperemia of the skin of the face and head, puffiness of the face. The skin is moist. Blood pressure - 19/min., BP - 125/80 mm Hg, HR=P8=104/min. What is the most likely diagnosis?

Тепловий удар легкого ступеня Mild heat stroke

Тепловий удар середнього ступеня Moderate heatstroke

Нейроциркуляторна дистонія Neurocirculatory dystonia

Тепловий удар важкого ступеня Severe heat stroke

Сонячний удар Sunstroke

70 / 200
Вагітна на останньому триместрі, прийнявши горизонтальне положення, знепритомніла. Черговий лікар підняв хвору, надавши їй вертикальне положення. Хвора опритомніла. Тони серця чисті, ритмічні, 76/хв., дихання везикулярне, АТ- 100/70 мм рт.ст. Який найбільш імовірний діагноз? A pregnant woman in the last trimester, having assumed a horizontal position, fainted. The doctor on duty raised the patient, giving her a vertical position. The patient fainted. Heart tones are clear, rhythmic, 76/min ., vesicular breathing, blood pressure - 100/70 mm Hg. What is the most likely diagnosis?

Стиснення нижньої порожнистої вени маткою Compression of the inferior vena cava by the uterus

Вегето-судинна дистонія Vegeto-vascular dystonia

Істерія Hysteria

Вазодепресивне зомління Vasodepressant grinding

Ортостатичний колапс Orthostatic Collapse

71 / 200
Хлопчик 12-ти років хворіє на атопічну бронхіальну астму з важким перебігом. Під час останнього нападу 4 інгаляції сальбутамолу ефекту не дали. Наросли задишка, тахікардія, неспокій. Шкіра бліда з ціанотичним відтінком. У легенях різко ослаблене дихання, хрипи не вислуховуються. Який з перелічених заходів є першочерговим? A 12-year-old boy suffers from atopic bronchial asthma with a severe course. During the last attack, 4 inhalations of salbutamol had no effect. Shortness of breath, tachycardia, restlessness developed. Skin she is pale with a cyanotic shade. Breathing is sharply weakened in the lungs, wheezing is not heard. Which of the listed measures is the first priority?

Інгаляція зволоженого кисню Inhalation of humidified oxygen

Внутрішньовенне введення кларитромі-цину Intravenous administration of clarithromycin

Внутрішньовенне введення еуфіліну Intravenous Euphilin

Внутрішньовенне введення амброксолу Intravenous administration of ambroxol

Внутрішньовенне введення преднізолону Intravenous administration of prednisone

72 / 200
Хворий скаржиться на головний біль, біль у ділянці серця, нудоту, сухість у роті, безсоння. Об’єктивно: обличчя гіперемоване, тремор пальців рук і язика, хода некоординована, виражена пітливість, настрій різко знижений. Зловживає спиртними напоями, останню добу алкоголь не вживав. Визначте психопатологічний синдром: The patient complains of a headache, heart pain, nausea, dry mouth, insomnia. Objectively: hyperemic face, tremors of fingers and tongue, gait uncoordinated, pronounced sweating, mood sharply lowered. Abuses alcoholic beverages, has not consumed alcohol in the last day. Define psychopathological syndrome:

Депресивний Depressed

Астенічний Asthenic

Деліріозний Delirious

Абстинентний Abstinent

Аментивний Amentary

73 / 200
У дівчини 19-ти років після зґвалтування з’явилися постійна плаксивість, тривога, настрій став пригніченим, знизилася працездатність, рухи загальмовані, виникли думки про недоцільність життя. Визначте психопатологічний стан: After the rape, a 19-year-old girl developed constant tearfulness, anxiety, her mood became depressed, her work capacity decreased, her movements were inhibited, and she had thoughts about the futility of life. Define psychopathological condition:

Тривожна депресія Anxious Depression

Реактивна депресія Reactive Depression

Ендогенна депресія Endogenous depression

Судинна депресія Vascular depression

Інволюційна депресія Involutional depression

74 / 200
Хвора 46-ти років скаржиться на появу протягом двох тижнів випорожнень кров’ю більш за 12 разів на добу, болю в животі, суглобах; болючі висипки у порожнині рота та нижніх кінцівках, набряк колінних та гомілковостопних суглобів, слабкість, підвищення температури тіла понад 39oC, втрату ваги тіла. Який метод обстеження є найбільш інформативним? A 46-year-old patient complains of the appearance of bloody stools more than 12 times a day, pain in the abdomen, joints, painful rashes in the oral cavity for two weeks and lower limbs, swelling of knee and ankle joints, weakness, increase in body temperature above 39oC, loss of body weight. Which examination method is the most informative?

Ендоскопічне дослідження Endoscopy examination

Рентгенологічне дослідження кишечнику X-ray examination of intestines

Рентгенографія суглобів X-ray of joints

Загальний аналіз крові General blood test

Біопсія кишечнику та шкіри Intestinal and skin biopsy

75 / 200
Сімейного лікаря, який іде у поїзді, запросили у сусіднє купе до пасажира, який, за словами оточуючих, поперхнувся шматком м’яса. Об’єктивно: ціаноз, при спробі вдихнути роздається різкий свист. Який з перерахованих заходів лікар має виконати перш за все? The family doctor, who is traveling on the train, was invited to the next compartment to a passenger who, according to those around him, had choked on a piece of meat. Objectively: cyanosis, when trying to inhale, a sharp whistle is heard. Which of the listed measures should the doctor perform first of all?

Прийом Геймліха Heimlich reception

- -

Зупинити поїзд та викликати швидку Stop the train and call an ambulance

Трахеотомія підручними засобами Tracheotomy with improvised means

Почати виконувати штучну вентиляцію легень Start CPR

76 / 200
Жінка 23-х років звернулась до лікаря зі скаргами на незначні кров’янисті виділення з піхви протягом доби, болі в низу живота. Остання нормальна менструація 2 місяці тому. Об’єктивно загальний стан не порушений. Живіт при пальпації м’який, болючий в нижніх відділах. Виділення кров’янисті незначні. Яке додаткове обстеження є доцільним? A 23-year-old woman consulted a doctor with complaints of minor vaginal bleeding during the day, pain in the lower abdomen. The last normal menstruation was 2 months ago. Objectively, the general condition is not disturbed. The abdomen is soft on palpation, painful in the lower parts. Blood discharge is insignificant. What additional examination is appropriate?

Ультразвукове обстеження Ultrasound examination

Імунологічний тест на вагітність Immunological pregnancy test

Рентгенологічне обстеження X-ray examination

Пункція заднього склепіння піхви Puncture of the posterior vault of the vagina

Лапароскопічне обстеження Laparoscopic examination

77 / 200
Чоловіка 39-ти років доставлено до лікарні з ножовим пораненням у живіт. Об’єктивно: шкіра звичайного кольору, Ps-112/хв., ритмічний, АТ- 120/90 мм рт.ст. На передній черевній стінці рана 1,5х2,5 см в правому підребер’ї. Живіт при пальпації помірно болючий в ділянці рани, перистальтика вислуховується, симптомів подразнення очеревини немає. Яка подальша тактика? A 39-year-old man was brought to the hospital with a stab wound to the abdomen. Objectively: skin of normal color, Ps-112/min., rhythmic, BP- 120/90 mm Hg. There is a 1.5x2.5 cm wound in the right hypochondrium on the front abdominal wall. The abdomen is moderately painful on palpation in the area of ​​the wound, peristalsis is heard, there are no symptoms of peritoneal irritation. What are the next tactics?

Лапароцентез Laparocentesis

Оглядова рентгенографія органів черевної порожнини Overview x-ray of abdominal organs

Діагностична лапаротомія Diagnostic laparotomy

Ревізія та первинна хірургічна обробка рани Revision and primary surgical treatment of the wound

Лапароскопія Laparoscopy

78 / 200
Хлопчик 6-ти років знаходиться в реанімаційному відділенні з приводу олігоануричної стадії гострої ниркової недостатності. При ЕКГ-моніторингу зафіксовано шлуночкову фібриляцію. Які реанімаційні заходи слід провести першочергово? A 6-year-old boy is in the intensive care unit due to the oligoanuric stage of acute renal failure. Ventricular fibrillation was detected during ECG monitoring. What resuscitation measures should be performed as a priority?

Штучна вентиляція легень Artificial lung ventilation

Введення кальцію хлориду Introduction of calcium chloride

Введення розчину адреналіну Injection of adrenaline solution

Електродефібриляція Electrodefibrillation

Проведення оксигенотерапії Carrying out oxygen therapy

79 / 200
Хвора 58-ми років перебуває в інфекційному відділенні з приводу важкої форми вірусного гепатиту A. На 4-й день лікування стан різко погіршився, з’явились сонливість, дезорієнтація у просторі, нечітка мова, тонічні судоми. У крові: ШЗЕ- 50 мм/год, ер.- 2, 7 • 1012/л, загальний білірубін - 300 мкмоль/л, АсАТ- 2,9 ммоль/год л, АлАТ- 3,1ммоль/год л. Яке найбільш імовірне ускладнення виникло? A 58-year-old patient is in the infectious department due to a severe form of viral hepatitis A. On the 4th day of treatment, the condition worsened sharply, drowsiness, disorientation appeared in space, slurred speech, tonic convulsions. In the blood: SZE - 50 mm/h, ER - 2.7 • 1012/l, total bilirubin - 300 μmol/l, AsAT - 2.9 mmol/h l, AlAT - 3.1 mmol/h l. What is the most likely complication that occurred?

Геморагічний інсульт Hemorrhagic stroke

Тромбоемболія легеневої артерії Thromboembolism of the pulmonary artery

Синдром Бадда-Кіарі Budd-Chiari syndrome

Уремічна кома Uremia Coma

Гостра печінкова енцефалопатія Acute hepatic encephalopathy

80 / 200
Хворий 58-ми років доставлений до приймального відділення з діагнозом гострий інфаркт міокарда. Раптово знепритомнів. Об’єктивно: пульс не визначається, дихання відсутнє, зіниці розширені. Яка тактика чергового лікаря? A 58-year-old patient was brought to the emergency room with a diagnosis of acute myocardial infarction. He suddenly fainted. Objectively: the pulse is not detected, there is no breathing, the pupils are dilated. What tactics of the doctor on duty?

Розпочати серцево-легеневу реанімацію Start CPR

Надання невідкладної медикаментозної допомоги Providing emergency medical care

Викликати чергову реанімаційну бригаду Call another resuscitation team

Негайна реєстрація ЕКГ для визначення подальшої тактики Immediate ECG registration to determine further tactics

Негайне транспортування хворого до відділення реанімації Immediate transportation of the patient to the intensive care unit

81 / 200
У хлопчика 6-ти років, хворого на гемофілію А, після травми виник гострий біль у правому колінному суглобі. Яка невідкладна допомога на догоспітальному етапі? A 6-year-old boy with hemophilia A developed acute pain in the right knee joint after an injury. What is the emergency care at the pre-hospital stage?

Введення антибіотиків Introduction of antibiotics

Накладання стискаючої пов’язки Applying compression bandage

Введення знеболюючих засобів Injection of painkillers

Накладання іммобілізаційної шини Immobilization splint

Кріопреципітат 15-20 ОД АГГ на 1 кг маси тіла Cryoprecipitate 15-20 units of AGG per 1 kg of body weight

82 / 200
У дівчинки 9-ти років на фоні отруєння грибами відмічається вкрай тяжкий загальний стан. Шкірні покриви різко бліді, свідомість затьмарена. Протягом останніх 2-х діб спостерігається анурія на фоні інтенсивної консервативної терапії. Яка подальша лікувальна тактика? A 9-year-old girl has an extremely severe general condition against the background of mushroom poisoning. The skin is sharply pale, consciousness is clouded. During the last 2 days, anuria has been observed on against the background of intensive conservative therapy. What are the further treatment tactics?

Провести гемосорбцію Perform hemosorption

Продовжити призначену терапію Continue prescribed therapy

Провести гемодіаліз Perform hemodialysis

Провести плазмаферез Perform plasmapheresis

Посилити консервативну терапію Strengthen conservative therapy

83 / 200
У жінки в 37 тижнів вагітності виникла кровотеча із статевих шляхів, наступила втрата свідомості. Про що слід подумати в першу чергу, виходячи з цих даних? A woman at 37 weeks of pregnancy started bleeding from the genital tract, lost consciousness. What should you think about first, based on these data?

Геморагічний шок Hemorrhagic shock

Рак шийки матки Cervical cancer

Центральне передлежання плаценти Central placenta previa

Септичний шок Septic shock

Відшарування плаценти Detachment of the placenta

84 / 200
У хворої була проведена операція екстирпації матки без додатків. Через 4 години після операції з’явилася цегляного кольору сеча, болі внизу живота, хвора у свідомості. Який найбільш імовірний діагноз? The patient underwent an operation to extirpate the uterus without appendages. 4 hours after the operation, brick-colored urine appeared, pain in the lower abdomen, the patient is conscious. What is the most likely diagnosis?

Інтраопераційне пошкодження сечового міхура Intraoperative bladder injury

Кіста нирки Kidney cyst

Рак нирки Kidney cancer

Гострий гломерулонефрит Acute glomerulonephritis

Нирковокам’яна хвороба, ниркова коліка Kidney stone disease, renal colic

85 / 200
Хвора 25-ти років внаслідок ДТП отримала важку черепно-мозкову травму, знаходиться в коматозному стані. Проведена М-Ехо-енцефалографія. Знайдено зміщення серединних структур мозку вправо на 12 мм. За допомогою комп’ютерної томографії в лобно-тім’яній ділянці зліва знайдена субдуральна гематома розміром 6х7х7 см. Яка тактика лікування? A 25-year-old female patient received a severe brain injury as a result of a traffic accident, is in a comatose state. An M-Echo encephalography was performed. A shift of the middle brain structures to the right was found 12 mm. With the help of computer tomography, a subdural hematoma measuring 6x7x7 cm was found in the fronto-parietal area on the left. What are the treatment tactics?

Хірургічне лікування Surgical treatment

Проведення краніоцеребральної гіпотермії Craniocerebral hypothermia

Нагляд за хворою у динаміці Monitoring the patient in dynamics

Застосування оксібаротерапії Use of oxybarotherapy

Проведення дезінтоксикаційної терапії Carrying out detoxification therapy

86 / 200
Хворий 48-ми років надійшов до стаціонару із клінікою печінкової недостатності. При огляді виявлено незначні неврологічні зрушення. Втрата ваги незначна. Наявний асцит легко піддається лікуванню. У крові: білірубін в сироватці крові -3,0 мг/%, рівень альбуміну - 3,2 мг/%, про-тромбіновий індекс - 75%. До якої групи печінково-клітинної недостатності можна віднести дані зрушення згідно з класифікацією печінкової дисфункції за Чайлд-П’ю? A 48-year-old patient was admitted to a hospital with a liver failure clinic. Examination revealed minor neurological changes. Weight loss is minor. Existing ascites is easily treatable. In the blood: bilirubin in blood serum -3.0 mg/%, albumin level - 3.2 mg/%, pro-thrombin index - 75%. To which group of hepatocellular insufficiency can these shifts be attributed according to the classification of liver dysfunction according to Child-P 'Yu?

2 2

1 б 1 b

1 а 1 а

3 3

1 1

87 / 200
У хворої 53-х років із оклюзією правої нирки каменем та загостренням хронічного пієлонефриту розвинувся інфекційно-токсичний шок. Призначена інфузійна терапія. Після підвищення артеріального тиску з метою профілактики повторного зниження його, які слід здійснити першочергові заходи? A 53-year-old patient with occlusion of the right kidney by a stone and exacerbation of chronic pyelonephritis developed infectious-toxic shock. Infusion therapy was prescribed. After increasing blood pressure in order to prevent repeated reducing it, what priority measures should be taken?

Призначити серцеві глікозиди Prescribe cardiac glycosides

Відновити відтік сечі з нирки Restore the outflow of urine from the kidney

Призначити сечогінні препарати Prescribe diuretics

Призначити гідрокарбонат натрію Prescribe sodium bicarbonate

Посилити антибактеріальну терапію Strengthen antibacterial therapy

88 / 200
У дитини 10-ти років зупинена артеріальна кровотеча (плечова артерія) через 10 хвилин після травми. При надходженні АТ- 90/50 мм рт.ст, еритроцити - 2, 5 • 1012/л, Hb- 60 г/л. Раніше при переливанні цільної крові була анафілактоїдна реакція. Який препарат крові найбільш безпечний для корекції крововтрати? A 10-year-old child has stopped arterial bleeding (brachial artery) 10 minutes after the injury. At admission, blood pressure is 90/50 mmHg, erythrocytes - 2, 5 • 1012/l, Hb- 60 g/l. Previously, there was an anaphylactoid reaction during transfusion of whole blood. Which blood preparation is the safest for blood loss correction?

Цільна кров Whole blood

Лейкоконцентрат Leukoconcentrate

Еритроцитарна маса Erythrocyte mass

Відмиті еритроцити Washed erythrocytes

Свіжозаморожена плазма Fresh-frozen plasma

89 / 200
Дитину 11-ти місяців на 3-й день хвороби госпіталізовано до інфекційного стаціонару з підвищенням температури тіла до 38oC, багаторазовим блюванням і частими водянистими випорожненнями. Маса тіла знижена на 6%. Який метод регідратації треба призначити? On the 3rd day of illness, an 11-month-old child was hospitalized in an infectious disease hospital with an increase in body temperature to 38oC, repeated vomiting and frequent watery stools. Body weight decreased by 6%. What method of rehydration should be prescribed?

Введення рідини ендогастрально Endogastric fluid administration

Введення рідини підшкірно крапельно Subcutaneous drip

Внутрішньовенний крапельний Intravenous drip

Внутрішньовенний струминний Intravenous jet

Оральна регідратація Oral rehydration

90 / 200
Хворий 66-ти років скаржиться на серцебиття, болі у серці, слабкість. Симптоми з’явилися зненацька, після фізичного навантаження. На ЕКГ: комплекси QRS поширені до 0,12 секунд, зубець Р перед шлуночковим комплексом відсутній, ЧСС-200/хв. Який препарат є препаратом вибору в цьому випадку? A 66-year-old patient complains of palpitations, heart pain, weakness. The symptoms appeared suddenly, after physical exertion. On the ECG: QRS complexes spread to 0 ,12 seconds, the P wave before the ventricular complex is absent, HR-200/min. What drug is the drug of choice in this case?

Верапаміл Verapamil

Лідокаїн Lidocaine

Обзидан Обзидан

Хінідин Quinidine

Дигоксин Digoxin

91 / 200
Хвора на гіпертиреоз 32-х років самостійно відмінила прийом антитиреоїдних засобів. Скаржиться на серцебиття, лихоманку до 38,5oC, болі у животі, пронос, блювання. Об’єктивно: психомоторне збудження, ЧСС- 140/хв., АТ- 150/90 мм рт.ст. Який патологічний стан розвинувся у хворої? A 32-year-old patient with hyperthyroidism stopped taking antithyroid drugs on her own. She complains of palpitations, fever up to 38.5oC, abdominal pain, diarrhea, vomiting. Ob' objectively: psychomotor excitement, heart rate - 140/min., blood pressure - 150/90 mm Hg. What pathological condition has developed in the patient?

Гіпертензивний криз Hypertensive crisis

Тиреотоксичний криз Thyrotoxic crisis

Гостра надниркова недостатнїсть Acute adrenal insufficiency

Криз при феохромоцитомї Pheochromocytoma crisis

Вегето-судинний криз Vegetovascular crisis

92 / 200
На місці ДТП лїкар ШМД оглядає дівчинку 12-ти років. Виявлено ознаки порушеної перфузії: мармуровість шкірних покривів, уповільнена перфузія нігтьових лож, ціаноз носо-губного трикутника, ЧСС- 140/хв, систолічний тиск 40 мм рт.ст. Який метод є оптимальним для корекції ге-модинамічних розладів? At the scene of the accident, the SMD doctor examines a 12-year-old girl. Signs of impaired perfusion were detected: marbling of the skin, slowed perfusion of the nail beds, cyanosis of the nasolabial triangle, heart rate - 140/min, systolic pressure 40 mm Hg. Which method is optimal for correcting hemodynamic disorders?

Призначення атропіну Prescription of atropine

ефузія добутаміну dobutamine effusion

Призначення спазмолітичних препаратів Prescription of antispasmodic drugs

Синхронізована кардіоверсія Synchronized cardio version

Ефузія сольових розчинів Effusion of salt solutions

93 / 200
Пацієнтка 58-ми років, що знаходиться на стаціонарному лікуванні в терапевтичному відділені з приводу гіпертонічної хвороби, раптово знепритомніла. Черговий лікар діагностував зупинку серцевої діяльності та дихання. Після забезпечення прохідності дихальних шляхів необхідні наступні заходи: A 58-year-old female patient, who is undergoing inpatient treatment in the therapeutic department for hypertension, suddenly fainted. The doctor on duty diagnosed cardiac and respiratory arrest. After providing patency of the respiratory tract, the following measures are necessary:

Ввести внутрішньосерцево атропін Enter atropine intracardiacally

Ввести внутрішньосерцево адреналін Inject adrenaline intracardiacally

Штучна вентиляція легень, зовнішній масаж серця Artificial lung ventilation, external heart massage

Ввести внутрішньосерцево норадреналін Inject norepinephrine intracardiacally

Ввести внутрішньовенно лідокаїн Inject lidocaine intravenously

94 / 200
Жінка 34-х років після зґвалтування висловлює суїцидальні думки, скаржиться на зниження настрою. В даному випадку необхідно: A 34-year-old woman after rape expresses suicidal thoughts, complains of low mood. In this case, it is necessary:

Рекомендувати звернутися до судово-медичної експертизи Recommend to contact forensic medical examination

Ввести 2,0 сибазону Enter 2.0 sibzone

Рекомендувати звернутися до гінеколога Recommend to consult a gynecologist

Рекомендувати звернутися до терапевта Recommend to see a therapist

Здійснити госпіталізацію до психіатричної клініки Admit to a psychiatric clinic

95 / 200
До приймального відділення доставлено чоловіка 30-ти років після втоплення у прісній воді, яке відбулося близько 40 хвилин тому і супроводжувалося клінічною смертю та успішною реанімацією. Чоловік у свідомості, при аускультації у легенях вислуховуються хрипи над всіма легеневими полями. Основний напрямок подальшої терапії: A 30-year-old man was brought to the emergency department after drowning in fresh water, which occurred about 40 minutes ago and was accompanied by clinical death and successful resuscitation. The man is conscious, upon auscultation in the lungs, rales are heard over all lung fields. The main direction of further therapy:

Дегідратація Dehydration

Введення глюкокортикоїдів Introduction of glucocorticoids

Введення інгібіторів протеолізу Introduction of proteolysis inhibitors

Киснетерапія Oxygen therapy

Відновлення ОЦК Recovery of BCC

96 / 200
У роділлі 35-ти років відбулися треті термінові нормальні пологи. Загальна крововтрата склала 400 мл, через 1 годину з піхви з’явилося 100 мл кров’янистих виділень із згортками. Які діагностичні заходи необхідно провести для уточнення діагнозу? A 35-year-old woman gave birth to a third urgent normal delivery. The total blood loss was 400 ml, after 1 hour 100 ml of bloody discharge appeared from the vagina coils. What diagnostic measures should be carried out to clarify the diagnosis?

Зондування порожнини матки Probing of the uterine cavity

Визначення висоти стояння дна матки Determination of the standing height of the uterine fundus

Огляд шийки матки в дзеркалах Examination of the cervix in mirrors

Загальний аналіз крові General blood test

Кюретаж порожнини матки Curettage of the uterine cavity

97 / 200
Хвора 54-х років з міомою матки і анемією (Hb- 52 г/л, Ht- 17%) скаржиться на головний біль, слабкість, запаморочення. Лікар призначив жінці трансфузію одногрупних відмитих еритроцитів. Яка основна мета гемотрансфузії в цьому випадку? A 54-year-old female patient with uterine fibroids and anemia (Hb- 52 g/l, Ht- 17%) complains of headache, weakness, dizziness. The doctor prescribed a transfusion of the same group of washed erythrocytes to the woman. What is the main purpose of hemotransfusion in this case?

Поживна Nutrient

Детоксикаційна Detox

Стимулююча Stimulating

Замісна Replaceable

Гемодинамічна Hemodynamic

98 / 200
Хворий 19-ти років захворів тиждень тому з появи слабкості, зниження апетиту, важкості в епігастрії; помітив темний колір сечі. Об’єктивно: стан задовільний, інтоксикація не виражена, активний, склери і шкіра субіктеричні, печінка еластичної консистенції, виступає на 2 см. У сироватці виявлений HBsAg, АлАТ- 2,12 ммоль/л год; білірубін 39,7 мкмоль/л. Де доцільно лікувати даного пацієнта? A 19-year-old patient became ill a week ago with the onset of weakness, decreased appetite, heaviness in the epigastrium; he noticed the dark color of the urine. Objectively: the condition is satisfactory, intoxication is not expressed, active, sclera and skin are subicteric, the liver has an elastic consistency, protrudes 2 cm. In the serum, HBsAg, AlAT - 2.12 mmol/l h; bilirubin 39.7 μmol/l. Where is it appropriate to treat this patient?

Інфекційне відділення Infectious disease department

Боксове відділення Boxing Department

Соматичне відділення Somatic Department

Амбулаторно Outpatient

Стаціонар на дому Inpatient at home

99 / 200
Жінка 47-ми років госпіталізована зі скаргами на слабкість, жовтяницю, свербіж шкіри. Захворіла 2,5 місяці тому, гостро, з явищами ознобу та підвищенням температури до 39 °C; через 2 тижні виникла наростаюча жовтяниця. Об’єктивно: значна жовтяниця, печінка не пальпується, жовчний міхур збільшений, не болючий. Білірубін крові - 190 мкмоль/л, за рахунок прямого. Кал ахолічний. Яка ймовірна причина жовтяниці у хворої? A 47-year-old woman was hospitalized with complaints of weakness, jaundice, itchy skin. She fell ill 2.5 months ago, acutely, with chills and a temperature rise to 39 °C; after 2 weeks, increasing jaundice appeared. Objectively: significant jaundice, the liver is not palpable, the gallbladder is enlarged, not painful. Blood bilirubin - 190 μmol/l, due to direct. Acholic stools. What is the probable cause of jaundice in the patient?

Гемолітична жовтяниця Hemolytic Jaundice

Хвороба Жильбера Gilbert's disease

Паренхіматозна жовтяниця Parenchymal jaundice

Механічна жовтяниця Mechanical jaundice

Синдром Каролі Caroli syndrome

100 / 200
Хворий 30-ти років скаржиться на пронос з домішками слизу і крові, переймоподібний біль у животі. Хворіє 4 роки, впродовж яких схуд на 10 кг. Об’єктивно: Ps-100/хв., АТ100/60 мм рт.ст., температура - 38oC. Живіт м’який, болючий у лівій здухвинній ділянці. У крові: ер.- 3,11*1012/л, НЬ-85 г/л, лейк.- 11,2х109/л, ШЗЕ- 32 мм/год. Реакція Грегерсена позитивна. Іригоскопія - товста кишка звужена, зернистість слизової оболонки, гаустри відсутні, контури нечіткі. Який найбільш імовірний діагноз? A 30-year-old patient complains of diarrhea with impurities of mucus and blood, spasm-like abdominal pain. He has been ill for 4 years, during which he lost 10 kg. Objectively : Ps-100/min., BP 100/60 mm Hg, temperature - 38oC. The abdomen is soft, painful in the left iliac region. In the blood: er.- 3.11*1012/l, НБ-85 g /l, leuk.- 11.2x109/l, SZE- 32 mm/h. Gregersen's reaction is positive. Irigoscopy - the colon is narrowed, the mucous membrane is granular, haustra are absent, the contours are unclear. What is the most likely diagnosis?

Синдром подразненої товстої кишки Irritable colon syndrome

Хвороба Крона Crohn's disease

Туберкульоз кишечнику Intestinal tuberculosis

Хронічний коліт Chronic Colitis

Неспецифічний виразковий коліт Nonspecific ulcerative colitis

101 / 200
У хворого 49-ти років діагностовано ішемічний інсульт. Прийнято рішення про початок тромболізісної терапії. В які строки від початку захворювання терапія буде максимально ефективною? A 49-year-old patient was diagnosed with an ischemic stroke. A decision was made to start thrombolysis therapy. At what time from the onset of the disease will the therapy be maximally effective?

До 1-ї доби Until the 1st day

В діапазоні 6-12 годин In the range of 6-12 hours

До 1-ї години Until the 1st hour

До 6-ти годин Up to 6 hours

До 3-х годин Up to 3 hours

102 / 200
До приймального відділення лікарні госпіталізовано хворого 25-ти років із політравмою. Діагностовано ЗЧМТ. Забій головного мозку І ст. Перелом кісток тазу, лівої стегнової кістки. З метою проведення масивної інфузійно-трансфузійної терапії хворому проведено пункцію та катетеризацію v. subclavia dextra центральним венозним катетером. Центральний венозний тиск у хворого негативний. Яке ускладнення можна отримати під час пункції центральної вени при негативному центральному венозному тиску? A 25-year-old patient with polytrauma was admitted to the hospital's admissions department. TBI was diagnosed. Cerebral contusion of the 1st degree. Fracture of the pelvis and left femur. In order to carry out of massive infusion-transfusion therapy, the patient underwent a puncture and catheterization of the v. subclavia dextra with a central venous catheter. The patient's central venous pressure is negative. What complications can occur during a central vein puncture with a negative central venous pressure?

Тромбоемболія Thromboembolism

Повітряна емболія Air embolism

Венозна кровотеча Venous bleeding

Інфекційні ускладнення Infectious complications

Жирова емболія Fat embolism

103 / 200
Хворий 38-ми років, що страждає на нирковий туберкульоз, привезений родичами до районної лікарні в непритомному стані. Об’єктивно: кома 1. Гіпотрофія 2 ст. Шкіра та слизові коричневого кольору з бронзовим відливом. На шкірі окремі осередки вітиліго. АТ- 65/0 мм рт.ст., ЧСС-44/хв. Діагностована гостра надниркова недостатність. Який препарат треба ввести в першу чергу? A 38-year-old patient suffering from renal tuberculosis was brought by relatives to the district hospital in an unconscious state. Objectively: coma 1. Hypotrophy of the 2nd stage. Skin and mucous membranes of brown color with a bronze tint. On the skin there are isolated foci of vitiligo. Blood pressure - 65/0 mm Hg, heart rate - 44/min. Acute adrenal insufficiency is diagnosed. What drug should be administered first?

Реополіглюкін Rheopoliglyukin

Новодрін Novodrin

Адреналін Adrenaline

Мезатон Мезатон

Дексаметазон Dexamethasone

104 / 200
Хвора 22-х років госпіталізована сімейним лікарем у відділення хірургії. Скарги на гострі болі у ділянці ануса, підвищення температури тіла до 37,8oC. Пальпаторно визначається пухлиноподібне утворення у ділянці 2-х годин умовного циферблату, розміром 2х3 см, різко болісне. Який найбільш імовірний діагноз? A 22-year-old patient was hospitalized by a family doctor in the department of surgery. Complaints of sharp pains in the anus area, an increase in body temperature to 37.8oC. A tumor-like formation in the the 2 o'clock area of ​​the conditional dial, measuring 2x3 cm, is sharply painful. What is the most likely diagnosis?

Гострий геморой Acute hemorrhoids

Тріщина прямої кишки Fissure of the rectum

Гострий сфінктерит Acute sphincteritis

Гострий парапроктит Acute paraproctitis

Рак прямої кишки Rectal cancer

105 / 200
Селективні а-адреноміметики недоцільно застосовувати для поновлення серцевої діяльності внаслідок: Selective α-adrenomimetics should not be used to restore cardiac activity due to:

Здатності підвищувати систолічний артеріальний тиск Ability to increase systolic blood pressure

Відсутності позитивного ефекту на міокард Absence of a positive effect on the myocardium

Значного підвищення потреби міокарда у кисні Significant increase in myocardial oxygen demand

Нездатності підвищувати діастолічний артеріальний тиск Inability to increase diastolic blood pressure

Вираженого аритмогенного ефекту Pronounced arrhythmogenic effect

106 / 200
Дівчина 26-ти років, з метою суїци-ду, прийняла 50 таблеток снодійного. Стан вкрай важкий: непритомна, зіниці широкі без реакції на світло. Дихання хрипле, 10/хв., за участю допоміжних м’язів, ціаноз. В легенях численні вологі хрипи. Ps-124/хв., поверхневий, АТ- 90/40 мм рт.ст. В чому буде полягати невідкладна допомога? A 26-year-old girl took 50 sleeping tablets with the intention of committing suicide. The condition is extremely serious: she is unconscious, her pupils are wide without reaction to light. Her breathing is hoarse, 10/min., with the participation of auxiliary muscles, cyanosis. There are numerous wet rales in the lungs. Ps-124/min., superficial, BP- 90/40 mm Hg. What will emergency care consist of?

Допоміжне дихання респіратором 'Амбу' Assisted breathing with an 'Ambu' respirator

Гіпербарична оксигенація Hyperbaric oxygenation

Стимуляція діурезу і компенсація плазмозамінниками Stimulation of diuresis and compensation with plasma substitutes

Інтубація трахеї Tracheal intubation

Туалет трахеобронхіального дерева Toilet of the tracheobronchial tree

107 / 200
Хвора 26-ти років, незаміжня, звернулась до лікаря жіночої консультації з скаргами на головний біль, який завжди супроводжує менструацію. З анамнезу з’ясовано, що головний біль виникає не тільки під час місячних, але й після перенапруження, а також при різкій зміні погодних умов. Ритм менструацій збережений: через 28-29 днів, по 3-4 дні, безболісні. При гінекологічному огляді патології не виявлено. Який найбільш імовірний діагноз? A 26-year-old patient, unmarried, turned to the doctor of the women's consultation with complaints of a headache, which always accompanies menstruation. From the anamnesis, it was found that the headache occurs not only during menstruation, but also after overexertion, as well as with a sudden change in weather conditions. The rhythm of menstruation is preserved: after 28-29 days, 3-4 days at a time, painless. During a gynecological examination, no pathology was found. What is the most likely diagnosis?

Енцефаліт Encephalitis

Овуляторний синдром Ovulatory Syndrome

Гостре порушення мозкового кровообігу Acute cerebrovascular accident

Передменструальний синдром Premenstrual syndrome

Мігрень Migraine

108 / 200
Військовослужбовець 20-ти років потрапив у осередок дії ОР Евакуйований до МПБ, введено підшкірно 4 мл 0,1% розчину атропіну. Скарги на загальну слабкість, спрагу, сухість у горлі, охриплість голосу, збудження, дратівливість. Шкіра обличчя гіперемована, зіниці розширені В легенях жорстке дихання. Ps- 112/хв, АТ- 140/90 мм рт.ст. Що стало причиною зазначеної картини? A 20-year-old military man got into the center of action of the OR. He was evacuated to the emergency room, 4 ml of 0.1% atropine solution was injected subcutaneously. Complaints of general weakness, thirst, dryness in the throat, hoarseness of the voice, excitement, irritability. The skin of the face is hyperemic, the pupils are dilated. Breathing is hard in the lungs. Ps- 112/min, BP- 140/90 mmHg. What caused the indicated picture?

Атропінізація організму Atropinization of the body

Ураження зарином Sarin exposure

Ураження дифосгеном Diphosgene exposure

Ураження зоманом Infected by soman

Передозування реактиваторів холінестерази Overdose of cholinesterase reactivators

109 / 200
Потерпілого 33-х років через 3 години після землетрусу доставлено до клініки. Об’єктивно: відірвана верхня права кінцівка, кукса під джгутом. АТ- 60/30 мм рт.ст., ЧСС140 /хв. Який ступінь шоку? A 33-year-old victim was brought to the clinic 3 hours after the earthquake. Objectively: the upper right limb was torn off, the stump was under a harness. BP- 60/30 mm Hg, heart rate 140/min. What is the degree of shock?

II II

V V

I I

III III

IV IV

110 / 200
У чоловіка 37-ми років після фізичного навантаження раптово виникає виражений біль у правому підребер’ї, нудота та блювання. Біль триває 5 хвилин та знов повторюється через 1 годину. Симптом Мерфі позитивний. Який попередній діагноз? A 37-year-old man suddenly develops severe pain in the right hypochondrium, nausea and vomiting after physical exertion. The pain lasts 5 minutes and recurs after 1 hour . Murphy's symptom is positive. What is the previous diagnosis?

Гострий гепатит Host hepatitis

Жовчна коліка Biliary colic

Хронічний холецистит Chronic cholecystitis

Гострий апендицит Acute appendicitis

Гострий панкреатит Hostry Pancreatitis

111 / 200
У хлопчика 11-ти років протягом 30-ти хвилин на фоні відсутності свідомості відмічаються напади судом у вигляді тонічного напруження м’язів обличчя, розгиначів кінцівок, яке змінюється короткими посмикуваннями різних м’язових груп тулуба і кінцівок з відхиленням очних яблук вгору і в бік. Який найбільш імовірний попередній діагноз? An 11-year-old boy has convulsions in the form of tonic tension of facial muscles, limb extensors, which is replaced by short twitching of various muscle groups of the trunk and limbs with deviation of the eyeballs up and to the side. What is the most likely preliminary diagnosis?

Судомна реакція Convulsive reaction

Судомний напад неуточненого генезу Seizure of unspecified genesis

Епілептичний синдром Epileptic syndrome

Генералізований епілептичний статус Generalized status epilepticus

Епілептична реакція Epileptic reaction

112 / 200
У дитини 5-ти років, що хвора на грип, спостерігається втрата свідомості. При люмбальній пункції спинномозкова рідина витікала частими краплями, цитоз - 5 клітин. Про що в першу чергу треба думати? A 5-year-old child with flu has lost consciousness. During a lumbar puncture, cerebrospinal fluid leaked out in frequent drops, cytosis - 5 cells. What about first of all, do you have to think?

Непритомність Fainting

Менінгіт Meningitis

Епілепсія Epilepsy

Колапс Collapse

Нейротоксикоз Neurotoxicosis

113 / 200
Пацієнт під час ковтання цукерки захрипів, схопився за шию, посинів і через 3 хвилини знепритомнів. Об’єктивно: свідомість відсутня, обличчя і шия ціанотичні, нерегулярні та непродуктивні рухи грудної клітки. На сонних артеріях - слабка пульсація. При пальцевій ревізії ротоглотки визначається округле стороннє тіло, змістити яке не вдається. Спроба змістити його при перекиданні пацієнта через коліно - невдала. Які подальші дії? While swallowing a candy, the patient snored, grabbed his neck, turned blue and fainted after 3 minutes. Objectively: consciousness is absent, the face and neck are cyanotic, irregular and unproductive movements of the chest. There is a weak pulsation on the carotid arteries. During a finger inspection of the oropharynx, a rounded foreign body is identified, which cannot be dislodged. An attempt to dislodge it while rolling the patient over the knee is unsuccessful. What are the further actions?

Транспортування до стаціонару Transportation to hospital

Крікотиреотомія Cricothyrotomy

Верхня трахеотомія Upper tracheotomy

Нижня трахеотомія Lower tracheotomy

Пункція трахеї Trache puncture

114 / 200
При ДТП було травмовано пішохода. Свідомість відсутня, АТ- 80/50 мм рт.ст. та прогресивно знижується, грудна клітка парадоксально здіймається, права нижня кінцівка у стегні ротована назовні. Який попередній діагноз? A pedestrian was injured in a road accident. He is unconscious, blood pressure is 80/50 mm Hg and is progressively decreasing, the chest rises paradoxically, the right lower limb in the thigh externally rotated. What is the previous diagnosis?

Забій головного мозку. Травматичний шок III ст. Cerebral contusion. Traumatic shock III century.

Закритий перелом правої стегнової кістки зі зміщенням. Флотуюча грудна клітка, гемопневмоторакс Closed fracture of the right femur with displacement. Floating chest, hemopneumothorax

Поєднана скелетна-торакальна травма тяжкого ступеня. Закритий перелом правої стегнової кістки зі зміщенням. Флотуюча грудна клітка. Травматичний шок III ст. Combined skeletal-thoracic injury of a severe degree. Closed fracture of the right femur with displacement. Floating chest. Traumatic shock III degree.

Закритий перелом правої стегнової кістки зі зміщенням. Флотуюча грудна клітка, забій головного мозку Closed fracture of the right femur with displacement. Floating chest, contusion of the brain

Флотуюча грудна клітка, гемопневмото-ракс. Забій головного мозку. Травматичний шок III ст. Floating chest, hemopneumo-rax. Cerebral contusion. Traumatic shock of the 3rd century.

115 / 200
Дитина від першої вагітності, термінових пологів, з дистоцією плечиків, народилася з вагою 4620 г, довжина 54 см. Оцінка за шкалою Апгар 8-8 балів. При народженні закричав відразу, реакція на огляд помірно знижена. Крик гучний, роздратований. Через 30 хвилин після народження дитина стогне, з’явилися дихальні розлади, через годину тонічні судоми. Який попередній діагноз? A child from the first pregnancy, term delivery, with dystocia of the shoulders, was born weighing 4620 g, length 54 cm. Apgar score 8-8 points. At birth screamed immediately, the reaction to the examination is moderately reduced. The cry is loud, irritated. 30 minutes after birth, the child moans, respiratory disorders appeared, after an hour, tonic convulsions. What is the preliminary diagnosis?

Вроджена вада серця Congenital heart defect

Сидром дихальних розладів Sydrome of respiratory disorders

Вроджена пневмонія Congenital pneumonia

Пологова травма Cover Injury

Асфіксія Asphyxia

116 / 200
Молодого чоловіка вкусила гадюка у ділянку стопи. Який з нижченаведених невідкладних заходів слід виконувати саме при укусі гадюки? A young man was bitten by a viper in the area of ​​the foot. Which of the following emergency measures should be taken when bitten by a viper?

Накладення стерильної пов’язки на рану Putting a sterile bandage on the wound

Вичавлювання залишків отрути з рани Squeezing the remaining poison from the wound

Обробка рани антисептиками Treat the wound with antiseptics

Іммобілізація ураженої кінцівки Immobilization of the affected limb

Накладання тугого джгута на кінцівку вище ділянки укусу Putting a tight tourniquet on the limb above the bite

117 / 200
Робітник 28-ми років лакофарбного виробництва при аварійній ситуації під час розвантаження фосгену відчув специфічний запах прілого сіна внаслідок несправності протигазу. Під наглядом 6 годин. Скарг, окрім відрази до тютюну, не пред’являє, тому від подальшого спостереження категорично відмовляється. Об’єктивно: шкіра та слизові оболонки рожевого кольору, ЧД- 22/хв., ЧСС- 60/хв. У крові: без патологічних змін. Вкажіть необхідний термін перебування постраждалого під наглядом лікаря: The 28-year-old worker of the paint industry during an emergency situation during the unloading of phosgene felt a specific smell of stale hay due to a malfunction of the gas mask. Under supervision for 6 hours. Complaints, except for disgust to tobacco, does not present, therefore further observation is categorically refused. Objectively: skin and mucous membranes are pink in color, BH - 22/min., heart rate - 60/min. In the blood: no pathological changes. Specify the required length of stay of the victim under the supervision of a doctor:

24 години 24 години

6-12 годин 6-12 hours

12-18 годин 12-18 hours

36 годин 36 hours

4-6 годин 4-6 hours

118 / 200
У хворого з зупинкою серця впродовж 40 хвилин проводиться закритий масаж серця та штучне дихання. Який показник найбільш імовірно свідчить про ефективність реанімаційних заходів? Closed cardiac massage and artificial respiration are performed on a patient with cardiac arrest for 40 minutes. Which indicator most likely indicates the effectiveness of resuscitation measures?

Звуження зіниць Pupillary Constriction

Аускультативно прослуховуються серцеві тони Heart sounds are auscultated

Наявність пульсу на периферичних артеріях Presence of pulse on peripheral arteries

Поява самостійного дихання Emergence of independent breathing

Артеріальний тиск Blood pressure

119 / 200
На вулиці знайдена дитина 6-ти років без свідомості, яка тримає в руці обірваний електричний дріт. Об’єктивно: частота дихання 5/хв., скорочення серця аритмічні, тони глухі, пульс слабкого наповнення, ЧСС - 20/хв. Яку допомогу необхідно надати першочергово? A 6-year-old unconscious child was found on the street, holding a broken electric wire in his hand. Objectively: respiratory rate 5/min, heart contractions are arrhythmic , dull tones, weak pulse, heart rate - 20/min. What help should be provided as a priority?

Закритий масаж серця Closed heart massage

Призупинити вплив електричного струму Stop electric current

Штучне дихання Artificial respiration

Внутрішньосерцеве введення адреналіну Intracardiac administration of epinephrine

Відновити прохідність дихальних шляхів Restore airway patency

120 / 200
У хворого 36-ти років була проведена корекція анемії трансфузією 350 мл еритроцитарної маси. Через 25 хвилин після переливання з’явились скарги на задишку, нудоту, пропасницю, біль за грудниною та в поперековій ділянці. Об’єктивно: стан тяжкий, збуджений, задишка. АТ- 90/70 мм рт.ст. ЧСС- 112/хв. Тони приглушені, ритмічні. В сечі через 2 години виявлені жовчні пігменти. Яке ускладнення трансфузійної терапії виникло? A 36-year-old patient underwent correction of anemia by transfusion of 350 ml of erythrocyte mass. 25 minutes after the transfusion, complaints of shortness of breath, nausea, dyspnea, pain appeared behind the sternum and in the lumbar region. Objectively: the condition is severe, excited, shortness of breath. Blood pressure - 90/70 mm Hg. Heart rate - 112/min. Tones are muffled, rhythmic. Bile pigments were found in the urine after 2 hours. What complication transfusion therapy occurred?

Гостра гемолітична реакція Acute hemolytic reaction

Інфекційні ускладнення Infectious complications

Пірогенна реакція Pyrogenic reaction

Цитратна інтоксикація Citrate intoxication

Анафілактична реакція Anaphylactic reaction

121 / 200
Хворий 40-ка років скаржиться на інтенсивні болі в горлі при ковтанні, виражену задишку, що посилюється у горизонтальному положенні і супроводжується нападами задухи. При непрямій ларингоскопії виявлений абсцес надгортанника. Нижче розташовані відділи гортані оглянути не вдається. Від трахеотомії хворий категорично відмовився. На висоті чергового нападу задухи з’явився ціаноз, судоми та наступила зупинка дихання. Яку першочергову допомогу слід надати? A 40-year-old patient complains of intense pain in the throat when swallowing, pronounced shortness of breath, which worsens in a horizontal position and is accompanied by attacks of suffocation. During indirect laryngoscopy, an epiglottis abscess was detected . It is not possible to examine the parts of the larynx located below. The patient categorically refused tracheotomy. At the height of another attack of suffocation, cyanosis appeared, convulsions and respiratory arrest occurred. What first aid should be provided?

Штучна вентиляція легенів методом рот в рот Artificial lung ventilation by the mouth-to-mouth method

Конікотомія Conicotomy

Ендоларингеальне розкриття абсцесу під контролем прямої ларингоскопії Endolaryngeal abscess opening under the control of direct laryngoscopy

Трахеотомія Tracheotomy

Інтубація трахеї Tracheal intubation

122 / 200
Хворому 39-ти років необхідне переливання еритроцитарної маси. При визначенні групової належності крові пацієнта за системою еритроцитарних антигенів АВ0 аглютинація спостерігалась в краплях з сироватками 0 (І) та В (III). Кров якої групи потрібно перелити пацієнту? A 39-year-old patient needs an erythrocyte mass transfusion. When determining the patient's blood group according to the erythrocyte antigen system AB0, agglutination was observed in drops with sera 0 (I) and B (III). Blood of which group should be transfused to the patient?

- -

А (II) А (II)

АВ (IV) АВ (IV)

В (III) В(III)

0 (I ) 0 (I )

123 / 200
Хвора 37-ми років скаржиться на сильні болі у лівому боці, що заважають дихати, субфебрилітет. Захворіла після перебування на холоді з протягом. На другий день на боці з’явилася висипка. Який діагноз найбільш імовірний? A 37-year-old patient complains of severe pain in the left side that interferes with breathing, low-grade fever. She fell ill after being in the cold with a runny nose. On the second day, on the side with 'a rash appeared. What is the most likely diagnosis?

Абсцес легенів Lung abscess

Вірусна інфекція Herpeszoster Herpeszoster virus infection

Пневмонія Pneumonia

Сухий плеврит Dry pleurisy

Ексудативний плеврит Exudative pleurisy

124 / 200
До лікаря звернувся пацієнт 26-ти років, який працює конюхом. Хворіє 3-й день. to- 40oC. На лівій долоні виразка, не болюча, з темно-коричневим дном, набряклими краями і кров’янистими виділеннями. Навколо неї дочірні везикули з прозорим вмістом, виражений драглистий набряк. Який попередній діагноз? A 26-year-old patient who works as a groom came to the doctor. He has been ill for the 3rd day. to- 40oC. On the left palm, there is an ulcer, not painful, with a dark - with a brown bottom, swollen edges and bloody secretions. Around it are daughter vesicles with transparent contents, marked gelatinous edema. What is the previous diagnosis?

Сибірка Anthrax

Еризипелоїд Erysipeloid

Бешиха Бешиха

Фурункульоз Furunculosis

Алергічний дерматит Allergic dermatitis

125 / 200
Хворий 40-ка років захворів гостро, після повернення з Заїру. Скарги на високу температуру тіла, головний біль, блювання 'кавовою гущею', біль у м’язах та попереку. Об’єктивно: to тіла - 39,9oC, обличчя гіперемоване. Марить. Склери і шкіра жовті, петехіальний висип. Печінка +3 см, болюча. У крові: лейк.- 2 • 109/л, тромб.-45 • 109/л. Який попєрєдній діагноз? A 40-year-old patient became acutely ill after returning from Zaire. Complaints of high body temperature, headache, vomiting of 'coffee grounds', muscle pain and lower back. Objectively: body temperature - 39.9oC, hyperemic face. Daydreaming. Yellow sclera and skin, petechial rash. Liver +3 cm, painful. In blood: leuk.- 2 • 109/l, thrombocyte - 45 • 109/l. What is the preliminary diagnosis?

Жовта лихоманка Yellow fever

Вірусний гепатит В Viral hepatitis B

Малярія Malaria

Лептоспіроз Leptospirosis

Грип Грип

126 / 200
У хлопчика 15-ти років раптово підвищилася температура тіла до 39,5°C, виник біль у горлі під час ковтання. На другу добу на обличчі, шиї, верхній частині тулуба на тлі гіперемованої шкіри тулуба виник дрібнокрапковий розеольозний висип з блідим носогубним трикутником. Який найбільш імовірний діагноз? A 15-year-old boy had a sudden rise in body temperature to 39.5°C, a sore throat while swallowing. On the second day, his face, neck, on the upper part of the body, on the background of hyperemic skin of the body, a small dot roseolosis rash with a pale nasolabial triangle appeared. What is the most likely diagnosis?

Скарлатина Scarlatina

Краснуха Krasnukha

Інфекційний мононуклеоз Infectious mononucleosis

Кір Кір

Вітряна віспа Chicken Pox

127 / 200
Жінка 38-ми років впродовж 23-х років страждає на часті, серійні генералізовані судомні напади. Прогресує зниження пам’яті, інертність мислення, гарячність. 12 годин тому розпочалися генералізовані тоніко-клонічні напади. Впродовж останніх двох годин між нападами свідомість не поновлюється. Який першочерговий захід надання допомоги? A 38-year-old woman has been suffering from frequent, serial generalized convulsive seizures for 23 years. Progressive memory loss, mental inertia, fever. 12 hours ago generalized tonic-clonic seizures have begun. During the last two hours between seizures, consciousness does not return. What is the first aid measure?

Введення внутрішньовенно діазепаму Intravenous diazepam

Введення внутрішньом’язово магнію сульфату Intramuscular administration of magnesium sulfate

Введення внутрішньовенно натрію окси-бутирату Intravenous administration of sodium oxybutyrate

Введення внутрішньом’язово гексеналу Intramuscular injection of hexanal

Введення в клізмі хлоралгідрату Chloral hydrate enema

128 / 200
До лікарні доставлена жінка 50-ти років, яку 2 години тому покусав собака. Який оптимальний обсяг первинної хірургічної обробки укушеної рани? A 50-year-old woman who was bitten by a dog 2 hours ago was brought to the hospital. What is the optimal amount of primary surgical treatment of a bite wound?

Промивання рани розчинами антисептиків, що мають кисле середовище Wound washing with solutions of antiseptics that have an acidic environment

Дренування рани Wound drainage

Розсічення рани по ходу ранового каналу Wound dissection along the wound channel

Висічення рани в межах здорової тканини Wound excision within healthy tissue

Промивання рани розчинами антисептиків, що мають лужне середовище Wound washing with solutions of antiseptics that have an alkaline environment

129 / 200
У потерпілого 36-ти років, що переніс закриту черепно-мозкову травму, в процесі проведення ехо-енцефалоскопії виявлено зміщення серединного еха на 5 мм. Про що це свідчить? In a 36-year-old victim who suffered a closed craniocerebral injury, during the echo-encephaloscopy, a 5 mm shift of the median echo was found. What does this indicate ?

Наявність підоболонкової гематоми Presence of submucosal hematoma

Травматичний субарахноїдальний крововилив Traumatic subarachnoid hemorrhage

Тріщина кісток склепіння черепа Crack of skull vault bones

Ехо-ЕС патології не виявила Echo-ES did not detect pathology

Перелом основи черепа Fracture of the base of the skull

130 / 200
На хімічному заводі в результаті аварії стався викид в повітря приміщення парів ціаністих сполук. У працівників, що знаходились в цеху, з’явились почуття дертя в горлі, оніміння губ, язика, різкий головний біль, нудота, блювання, біль у животі, почервоніння шкіри. Які найважливіші невідкладні заходи потрібно здійснити для збереження життя уражених? At the chemical plant, as a result of the accident, vapors of cyanide compounds were released into the air. Workers in the workshop felt a scratchy feeling in the throat, numbness of the lips , tongue, sharp headache, nausea, vomiting, abdominal pain, reddening of the skin. What are the most important emergency measures to be taken to save the life of the affected?

Ввести внутрішньовенно 10 мл 1% натрію нітриту Enter 10 ml of 1% sodium nitrite intravenously

Провести на місці аварії інфузію метиленового синього 50 мл 1% розчину на 5% розчині глюкози Conduct infusion of methylene blue 50 ml of 1% solution on 5% glucose solution at the accident site

Інгаляція кисню через носові катетери Inhalation of oxygen through nasal catheters

Забезпечити вдихання амілнітриту на місці аварії Ensure inhalation of amyl nitrite at the scene of the accident

Оксибаротерапія у відділенні токсикології, антидотна терапія під час транспортування Oxybarotherapy in the toxicology department, antidote therapy during transportation

131 / 200
Пацієнт 79-ти років знаходиться у ПІТ в ранньому післяопераційному періоді з приводу кишкової непрохідності. Хворому необхідно проводити інфузійну терапію, парентеральне харчування. Поверхнева венозна сітка погано розвинута. Яка тактика в плані забезпечення венозного доступу? A 79-year-old patient is in the intensive care unit in the early postoperative period due to intestinal obstruction. The patient needs infusion therapy, parenteral nutrition. The superficial venous network is poorly developed. What tactics in terms of ensuring venous access?

Відновити ентеральне харчування Restore enteral nutrition

Пункція та катетеризація підключичної вени Puncture and catheterization of the subclavian vein

Катетеризація порожнини серця Catheterization of the heart cavity

Відмінити інфузійну терапію та парентеральне харчування Cancel infusion therapy and parenteral nutrition

Вводити ліки підшкірно Inject medication subcutaneously

132 / 200
Дитина 3-х місяців, яка народилася від серопозитивної матері щодо ВІЛ-інфікування, оглянута дільничим педіатром. Стан дитини соматично та неврологічно відповідає віковому статусу. В зв’язку з необхідністю проведення календарного профілактичного щеплення дитина направлена на вакцинацію проти дифтерії, кашлюка та правця (АКДП). Як провести таке щеплення даній дитині? A 3-month-old child born to a seropositive mother with regard to HIV infection was examined by a district pediatrician. The child's somatic and neurological condition corresponds to the age status. In connection with the need for calendar preventive vaccination, the child is sent for vaccination against diphtheria, pertussis and tetanus (AKDP). How to administer such vaccination to this child?

Вакцинація АКДП проводиться за спеціальним календарем AKDP vaccination is carried out according to a special calendar

Вакцинація АКДП проводиться за стандартним календарем AKDP vaccination is carried out according to the standard calendar

- -

Вакцинація АКДП проводиться після уточнення ВІЛ-статусу AKDP vaccination is carried out after clarification of HIV status

Вакцинація АКДП не проводиться AKDP vaccination is not carried out

133 / 200
У хворого 26-ти років, що лікується в гастроентерологічному відділенні з приводу пептичної виразки 12-палої кишки, годину тому з’явились гострий біль у животі, блювання. Об’єктивно: живіт дошкопо-ді6ний, різко болючий у верхніх відділах. Яке дослідження доцільно провести хворому для встановлення причини ускладнення хвороби? A 26-year-old patient, who is being treated in the gastroenterology department for a peptic ulcer of the duodenum, developed acute abdominal pain and vomiting an hour ago. Objectively: the abdomen is pre-injured, sharply painful in the upper parts. What research should be conducted on the patient to establish the cause of the complication of the disease?

Аналіз крові клінічний Clinical blood analysis

Рентгеноскопія шлунково-кишкового тракту з барієм X-ray of the gastrointestinal tract with barium

Ультразвукове дослідження органів черевної порожнини Ultrasound examination of abdominal organs

Оглядова рентгеноскопія живота в положенні стоячи Rentoscopy of the abdomen in the standing position

Іригоскопія Irigoscopy

134 / 200
У новонародженої дитини на третю добу життя з’явилися ознаки шлунково-кишкової кровотечі (блювання з домішками крові, мелена), постгеморагічної анемії. З якого розчину слід розпочати парентеральне лікування геморагічного синдрому у дитини? On the third day of life, a newborn child showed signs of gastrointestinal bleeding (vomiting with blood impurities, melena), posthemorrhagic anemia. From which solution should parenteral treatment of hemorrhagic syndrome in a child?

Глюкозо-сольовий розчин Glucose-saline solution

Амінокапронова кислота Aminocaproic acid

Тромбоцитарна маса Platelet Mass

Реополіглюкін Rheopoliglyukin

Свіжозаморожена плазма Fresh-frozen plasma

135 / 200
У роділлі 25-ти років із вродженою вадою серця в І періоді пологів з’явився кашель, задишка, харкотиння, болі за грудниною, тахікардія. АТ- 90/60 мм рт.ст., to-36,8oC. Роділля займає вимушене сидяче положення. Який найбільш імовірний діагноз? A 25-year-old woman in labor with a congenital heart defect developed cough, shortness of breath, sputum, chest pain, tachycardia in the first period of labor. AT-90/ 60 mm Hg, to-36.8oC. The woman in labor takes a forced sitting position. What is the most likely diagnosis?

Розрив матки Rupture of uterus

Напад бронхіальної астми Bronchial asthma attack

Набряк легень Pulmonary edema

Інфаркт міокарда Myocardial infarction

Емболія навколоплідними водами Amniotic fluid embolism

136 / 200
Пацієнт під час вставання з ліжка у реанімаційному відділенні миттєво посинів і втратив свідомість. Об’єктивно: свідомість відсутня, обличчя і шия ціанотичні, нерегулярні та непродуктивні рухи грудної клітки. На сонних артеріях - пульсація не визначається, при аускультації - серцеві тони не вислуховуються. Які подальші дії лікаря? The patient, while getting out of bed in the intensive care unit, instantly turned blue and lost consciousness. Objectively: unconsciousness, cyanotic face and neck, irregular and unproductive chest movements . On the carotid arteries - pulsation is not determined, during auscultation - heart tones are not heard. What are the further actions of the doctor?

Закритий масаж серця Closed heart massage

Виклик реаніматолога Intensivist call

Дефібриляція Defibrillation

Відкритий масаж серця Open heart massage

Штучна вентиляція легенів Artificial lung ventilation

137 / 200
З вогнища радіаційної катастрофи до медичного пункту полку доставлено 15 потерпілих. Через 20-25 хвилин після вибуху стали відчувати різку слабкість, запаморочення, нудоту, з’явилось невгамовне блювання. Стан хворих тяжкий. Кволі, апатичні, на запитання відповідають з великою затримкою. ЧСС- 120/хв. з частими екстрасистолами, АТ- 70/30 мм рт.ст., ЧД- 28/хв. За даними індивідуального дозиметра доза отриманого випромінювання склала 8 Гр. Який найбільш імовірний діагноз? 15 victims were taken from the site of the radiation disaster to the regiment's medical center. 20-25 minutes after the explosion, they began to feel sharp weakness, dizziness, nausea, uncontrollable vomiting appeared The condition of the patients is serious. Weak, apathetic, they answer questions with a long delay. Heart rate - 120/min. with frequent extrasystoles, BP - 70/30 mmHg, BP - 28/min. According to the data of the individual dosimeter, the dose of radiation received was 8 Gy. What is the most likely diagnosis?

Гостра променева хвороба III ст. Acute radiation sickness of the III century.

Гостра променева хвороба І ст. Acute radiation sickness of the 1st century

Гостра променева хвороба IV ст. Acute radiation sickness of the IV century.

Гостра променева хвороба V ст. Acute radiation sickness of the 5th century.

Гостра променева хвороба II ст. Acute radiation sickness of the II century.

138 / 200
Вагітна в терміні 32 тижні страждає на міастенію. З’явились розлади дихання, тахікардія, психомоторне збудження, яке змінюється млявістю, апатією, парезом кишечнику та сфінктерів. Першочергові засоби невідкладної допомоги: A 32-week pregnant woman suffers from myasthenia gravis. Respiratory disorders, tachycardia, psychomotor excitement appeared, which is replaced by lethargy, apathy, paresis of the intestines and sphincters. First-line remedies emergency care:

Профілактика гіпоксії плоду Prevention of fetal hypoxia

Введення транквілізаторів Introduction of tranquilizers

Термінове розродження Urgent delivery

ШВЛ с застосуванням міорелаксантів курареподібної дії Ventilation with curare-like muscle relaxants

Призначення прозерину Proserin appointment

139 / 200
У хворого 44-х років через 7 діб після переливання крові з’явились жовтяниця і гарячка, АТ- 90/60 мм рт.ст., Ps- 100/хв., добовий діурез 200 мл. Про яке ускладнення йдеться? A 44-year-old patient developed jaundice and fever 7 days after blood transfusion, blood pressure - 90/60 mm Hg, Ps - 100 /min., daily diuresis 200 ml. What complication is it about?

Пізня гемолітична трансфузійна реакція, гостра ниркова недостатність Late hemolytic transfusion reaction, acute renal failure

rPBI rPBI

Гострий холецистит Hostry Cholecystitis

Гострий гепатит Host hepatitis

Пневмонія Pneumonia

140 / 200
Постраждалий доставлений з зони пожежі через 2 години після виходу з зони вогню. Скаржиться на головний біль, різь в очах і сльозотечу, відчуття дертя в горлі, кашель, шум у вухах, запаморочення. Було одноразове блювання, явища подразнення верхніх дихальних шляхів помітно зменшилися. При огляді збуджений, ейфоричний, шкіра обличчя набрякла, гіперемована, слизова оболонка глотки яскраво-червоного забарвлення. Ps- 82/хв., задовільненого наповнення, тони серця ослаблені, АТ- 130/100 мм рт.ст., ЧД- 28/хв., дихання ослаблене, температура тіла 37,2oC. Яка імовірна патологія у постраждалого? The victim was brought from the fire zone 2 hours after leaving the fire zone. He complains of a headache, watery eyes and tearing, a scratchy feeling in the throat, cough, noise in the ears, dizziness. There was a single vomiting, the symptoms of irritation of the upper respiratory tract have noticeably decreased. On examination, he is excited, euphoric, the skin of the face is swollen, hyperemic, the mucous membrane of the pharynx is bright red in color. Ps- 82/min., satisfied filling, heart tones are weakened , blood pressure - 130/100 mm Hg, blood pressure - 28/min, breathing is weak, body temperature 37.2oC. What is the probable pathology of the victim?

Комбіноване отруєння СО ціанідами з опіком дихальних шляхів Combined CO poisoning by cyanides with burns of the respiratory tract

Отруєння чадним газом, опік верхніх дихальних шляхів Poisoning with carbon monoxide, burn of the upper respiratory tract

- -

Отруєння ціаністими сполуками Poisoning with cyanide compounds

Отруєння чадним газом Monoxide poisoning

141 / 200
У хлопчика 11-ти років на фоні тяжкого перебігу ревматизму з включенням до терапії преднізолону раптово з’явилися болі в животі, диспептичні явища, кров у випорожненнях. Який додатковий метод найбільш доцільно провести для уточнення діагнозу? An 11-year-old boy suddenly developed abdominal pain, dyspeptic symptoms, blood in stools against the background of a severe course of rheumatism with the inclusion of prednisolone in therapy. What additional method is most expedient to carry out to clarify the diagnosis?

Бактеріологічні дослідження Bacteriological studies

Рентгенологічне дослідження ШКТ X-ray examination of the gastrointestinal tract

Фіброгастродуоденоскопія Fibrogastroduodenoscopy

Шлункове фракційне зондування Gastric fractional sounding

рН-мєтрія pH-metry

142 / 200
Дитині 7-ми років у реанімаційному відділенні надаються реанімаційні заходи у зв’язку з зупинкою серця. Після проведення інтубації та ШВЛ почервоніли шкірні покриви, з’явився пульс на великих судинах, ЧСС- 40/хв., АТ- 50/30 мм рт.ст. Яка подальша тактика? A 7-year-old child is receiving resuscitation measures in the intensive care unit due to cardiac arrest. After intubation and mechanical ventilation, the skin turned red, a pulse appeared on large vessels, heart rate - 40/min., blood pressure - 50/30 mm Hg. What is the next strategy?

Введення серцевих глікозидів Introduction of cardiac glycosides

Введення глюкокортикоїдів Introduction of glucocorticoids

Введення сольових розчинів Introduction of saline solutions

Введення розчину допаміну Injection of dopamine solution

Введення 0,1% розчину атропіну Introduction of 0.1% atropine solution

143 / 200
У пацієнта 48-ми років, який звернувся в лікарню, після пальпації живота виник сильний головний біль, почервоніння та тремтіння всього тіла. АТ- 240/120 - 260/130 мм рт.ст. Який препарат є препаратом вибору для надання невідкладної допомоги? A 48-year-old patient who went to the hospital developed a severe headache, redness and trembling of the whole body after palpation of the abdomen. BP- 240/120 - 260 /130 mm Hg. Which drug is the drug of choice for providing emergency care?

Каптоприл Captopril

Лазикс Лазикс

Фенігідин Phenigidine

Клофелін Clofelin

Фентоламін Phentolamine

144 / 200
Хворий 49-ти років, шофер, доставлений в клініку через 3 години після діагностованого інфаркту міокарда. Під час транспортування декілька разів втрачав свідомість, виникали судоми. Об’єктивно: стан хворого важкий. АТ- 100/50 мм рт.ст. ЧСС- 35/хв. На ЕКГ: повна атріовентри-кулярна блокада, елевація сегменту ST у V1-V4. З чого необхідно розпочати надання невідкладної допомоги? A 49-year-old patient, a driver, was brought to the clinic 3 hours after a diagnosed myocardial infarction. During transportation, he lost consciousness several times, had convulsions. Objectively : the patient's condition is serious. Blood pressure - 100/50 mm Hg. Heart rate - 35/min. On the ECG: complete atrioventricular block, ST segment elevation in V1-V4. What is the need to start providing emergency care?

Електрокардіостимуляція Electronic cardiostimulation

Призначення морфіну Morpine Prescription

Призначення допаміну Dopamine prescription

Дефібриляція Defibrillation

Призначення серцевих глікозидів Prescription of cardiac glycosides

145 / 200
Хворого доставлено після аварії. Спостерігається інтенсивна гематурія, прогресивне погіршання стану хворого, підозра на поєднане пошкодження нирок та органів черевної порожнини. Враховуючи симптоми, хворому показано: The patient was delivered after an accident. There is intense hematuria, progressive deterioration of the patient's condition, suspicion of combined damage to the kidneys and organs of the abdominal cavity. Taking into account the symptoms, the patient is shown:

Ретроградна пієлографія Retrograde pyelography

Артеріографія судин нирок Arteriography of kidney vessels

Негайне хірургічне втручання Immediate surgery

Радіоізотопна ренографія Radioisotope renography

Консервативна терапія Conservative therapy

146 / 200
Хворий 38-ми років протягом тривалого часу відчував зубний біль. Згодом з’явилися біль голови і ока, загальна слабкість, підвищилася температура тіла. Об’єктивно: хемоз кон’юнктиви, застійна ін’єкція очного яблука, екзофтальм, обмеження рухів очного яблука. В крові: лейкоцитоз, ШЗЕ- 24 мм/год. На рентгенограмі орбіт патології не виявлено. Який найбільш імовірний діагноз? A 38-year-old patient had a toothache for a long time. Later, headache and eye pain appeared, general weakness, body temperature rose. Objectively: chemosis conjunctivae, congestive injection of the eyeball, exophthalmos, restriction of eyeball movements. In the blood: leukocytosis, ESR - 24 mm/h. On the X-ray of the orbits, no pathology was detected. What is the most likely diagnosis?

Флегмона орбіти Phlegmon of the orbit

Ретробульбарний крововилив Retrobulbar hemorrhage

Запальний псевдотумор орбіти Inflammatory pseudotumor of the orbit

Целюліт орбіти Cellulitis of the orbit

Теноніт Tenonite

147 / 200
У дівчинки 10-ти років яка страждає на хронічний гломерулонефрит, змішану форму, стан раптово погіршився: АТ-145/95 мм рт.ст., ендогенний креатинін -1,056 ммоль/л, сечовина - 18,8 ммоль/л, калій плазми - 7,5 ммоль/л, діурез - відсутній. Який препарат із діуретичних препаратів треба призначити першочергово? A 10-year-old girl suffering from chronic glomerulonephritis, mixed form, the condition suddenly worsened: BP-145/95 mm Hg, endogenous creatinine - 1.056 mmol/l, urea - 18.8 mmol/l, plasma potassium - 7.5 mmol/l, diuresis - absent. Which diuretic drug should be prescribed first?

Аріфон Аріфон

Лазикс Лазикс

Еуфілін Euphilin

Гіпотіазид Hypotiazid

Спіронолактон Spironolactone

148 / 200
У госпіталізованої 13-річної дитини скарги на підвищення температури тіла до 37,6oC, прояви дрібноточкових та екхімо-зних висипань на шкірі, носової кровотечі. Скарги з’явилися через 6 днів після перенесеної вірусної інфекції. Об’єктивно: пальпуються підщелепні лімфовузли, безболісні, рухомі. У крові: лейк.- 11,6 • 109/л, ер.-4,51 • 1012/л, Hb- 123 г/л, КП- 0,81, тромб.-9• 109/л, п- 4%, с- 63%, м- 4%, л- 24%, е- 5%. ШЗЕ-15 мм/год. Який попередній діагноз? A hospitalized 13-year-old child complains of an increase in body temperature up to 37.6oC, small-point and ecchymotic rashes on the skin, nosebleeds. Complaints appeared 6 days after a viral infection. Objectively: palpable submandibular lymph nodes, painless, mobile. In blood: leuk.- 11.6 • 109/l, er.-4.51 • 1012/l, Hb- 123 g/ l, KP- 0.81, thromb.-9• 109/l, p- 4%, c- 63%, m- 4%, l- 24%, e- 5%. SZE-15 mm/h. What previous diagnosis?

Менінгококцемія Meningococcemia

Гостра лейкемія Acute leukemia

Тромбоцитопенічна пурпура Thrombocytopenic purpura

Геморагічний васкуліт Hemorrhagic vasculitis

Реактивний лімфаденіт підщелепних лімфовузлів Reactive lymphadenitis of the submandibular lymph nodes

149 / 200
У роділлі 24-х років у пологах раптом погіршилося серцебиття плоду, матка в постійному гіпертонусі, з’явилися значні кров’янисті виділення з піхви. Роділля зблідла, Ps- 130/хв., АТ- 80/50 мм рт.ст. Який найбільш імовірний діагноз? In a 24-year-old woman during childbirth, the fetal heartbeat suddenly worsened, the uterus was in constant hypertonicity, significant vaginal discharge appeared. The woman in labor turned pale, Ps - 130/min., BP - 80/50 mm Hg. What is the most likely diagnosis?

Розрив матки Rupture of uterus

Розрив шийки матки Rupture of the cervix

Передчасне відшарування плаценти Premature placental abruption

Клінічно вузький таз Clinically narrow pelvis

Гострий апендицит Acute appendicitis

150 / 200
Хворий 62-х років хворіє на хронічне обструктивне захворювання легенів. Об’єктивно: ядуха, ЧД- 34/хв., ортопное, акроці-аноз, дихання клокочуче, чути на відстані, під час кашлю виділяється пінисте харкотиння рожевого кольору у великій кількості, Ps- 120/хв., слабкого наповнення, АТ-80/50 мм рт.ст. Над легенями - маса різнокаліберних вологих хрипів. В яке відділення має бути госпіталізований хворий? A 62-year-old patient suffers from chronic obstructive pulmonary disease. Objectively: dyspnea, heart rate - 34/min., orthopnea, acrocy-anosis, gurgling breathing , can be heard at a distance, during coughing, foamy sputum of a pink color is released in large quantities, Ps- 120/min., weak filling, BP-80/50 mm Hg. Above the lungs - a mass of various caliber moist rales. In which department should he be hospitalized patient?

Пульмонологічне відділення Pulmonology Department

Загальнотерапевтичне відділення General therapeutic department

Хворий не потребує госпіталізації The patient does not need hospitalization

Відділення реанімації та інтенсивної терапії Department of intensive care and intensive care

Хірургічне відділення Surgical department

151 / 200
У дівчинки 2-х тижнів, що народилася з вагою 3 кг, маса тіла на момент огляду 2900 г, груди смокче мляво, м’язова гіпотонія, рефлекси періоду новонародженості пригнічені, тім’ячко западає, губи сухі, клітор збільшений, гіперпігментація сосків молочних залоз та статевих губ. Зважаючи на ризик критичного стану насамперед у дитини слід динамічно оцінювати: A 2-week-old girl who was born with a weight of 3 kg, body weight at the time of examination is 2900 g, breast sucks sluggishly, muscle hypotonia, period reflexes newborns are depressed, the crown is sunken, the lips are dry, the clitoris is enlarged, hyperpigmentation of the nipples of the mammary glands and labia. Taking into account the risk of a critical condition, the child should first of all be dynamically evaluated:

Неврологічний статус Neurological status

Вагову криву Weight curve

Рівень кортизолу в крові Blood cortisol level

Артеріальний тиск Blood pressure

Гідратацію і рівень електролітів Hydration and electrolyte level

152 / 200
Дитина народжена в терміні гестації 39 тижнів з масою 3500 г, довжиною 54 см. Загальний стан при народженні середньої тяжкості за рахунок гострої асфіксії. Після проведення заходів первинної реанімації новонароджених у дитини з’явилося самостійне дихання, ЧСС- 110/хв., але зберігається ціаноз шкірних покривів. Яка подальша дія лікаря-неонатолога у даній ситуації? The child was born at a gestation period of 39 weeks with a weight of 3500 g, a length of 54 cm. The general condition at birth was of moderate severity due to acute asphyxia. After carrying out primary resuscitation of newborns the child started breathing independently, heart rate - 110/min., but the cyanosis of the skin remains. What is the further action of the neonatologist in this situation?

Відсмоктування слизу з верхніх дихальних шляхів Suction of mucus from the upper respiratory tract

Киснева підтримка Oxygen support

Інтубація трахеї Tracheal intubation

Допоміжна вентиляція легенів Assisted lung ventilation

Штучний масаж серця Artificial heart massage

153 / 200
У ВПВ поступила повторновагітна в 36 тижнів вагітності, яка скаржиться на відсутність рухів дитини протягом доби. Серцебиття плоду не вислуховується. З анамнезу: попередня вагітність завершилася мертвонародженням в 36-37 тижнів. Жінка страждає на цукровий діабет протягом 10-ти років. При надходженні рівень глюкози в крові натще 10,8 ммоль/л. При УЗД в 33-34 тижні виявлено подвійний контур голівки і тулуба, багатоводдя, передбачувана маса плоду 3800 г. Що першочергово слід було виконати після УЗД з метою запобігання ускладнень, що виникли? A re-pregnant woman at 36 weeks of pregnancy was admitted to the hospital, who complains about the lack of movement of the child during the day. The heartbeat of the fetus is not heard. From the anamnesis: the previous pregnancy ended in stillbirth at 36 37 weeks. The woman has been suffering from diabetes for 10 years. On admission, the fasting blood glucose level is 10.8 mmol/l. During ultrasound at 33-34 weeks, a double contour of the head and trunk, polyhydramnios, estimated fetal weight of 3800 g were revealed. What should be done first and foremost after the ultrasound in order to prevent the complications that have arisen?

Провести КТГ Perform CTG

Проведення тесту толерантності до глюкози Glucose tolerance test

Госпіталізація в акушерський стаціонар Hospitalization in an obstetric hospital

Повторне УЗД через 2 тижні Repeat ultrasound in 2 weeks

Призначення консультації ендокринолога Endocrinologist consultation appointment

154 / 200
У хворого 72-х років раптово розвинулось блювання кров’ю з крововтратою до 1,0 л. Об’єктивно: АТ- 70/40 мм рт.ст., ЧСС-112/хв. ЦВТ- 0. Які інфузійні середовища першочергово треба застосувати для лікування? A 72-year-old patient suddenly developed vomiting of blood with blood loss up to 1.0 L. Objectively: blood pressure - 70/40 mm Hg ., heart rate-112/min. CVT- 0. What infusion media should be used primarily for treatment?

Кристалоїдні розчини Crystalloid solutions

Колоїдні розчини Colloidal solutions

Свіжозаморожена плазма Fresh-frozen plasma

Жирові емульсії Fat emulsions

Розчини глюкози Glucose solutions

155 / 200
Хлопчик 15-ти років під час кросу на уроці фізкультури раптово впав, знепритомнів, з’явилися тоніко-клонічні судоми. Шкіра бліда, зіниці розширені. Дихання відсутнє. Пульс на магістральних судинах визначити не вдається. Який найбільш імовірний діагноз? A 15-year-old boy suddenly fell down during cross-country skiing in physical education class, fainted, had tonic-clonic convulsions. The skin is pale, the pupils are dilated. There is no breathing. The pulse on the main vessels cannot be determined. What is the most likely diagnosis?

Зомління Grinding

Клінічна смерть Clinical death

Біологічна смерть Biological death

Колапс Collapse

Напад епілепсії Attack of epilepsy

156 / 200
На місці ДТП знайдений чоловік з ознаками черепно-мозкової травми. Свідомість порушена за типом глибокої коми. Дихання утруднене, 'хропляче' з втягуванням податливих місць грудної клітки. Яка першочергова допомога необхідна? A man with signs of a brain injury was found at the scene of the accident. Consciousness is impaired according to the type of deep coma. Breathing is difficult, 'snoring' with pulling in the pliable parts of the chest. What first aid is necessary?

Забезпечити прохідність верхніх дихальних шляхів Ensure patency of the upper respiratory tract

Ввести в/в 40% розчин глюкози Enter IV 40% glucose solution

Ввести в/в розчин кордіаміну Inject cordiamine IV solution

Ввести в/в розчин налоксону Inject intravenous naloxone solution

Ввести в/м розчин промедолу Enter i/m retail promedol

157 / 200
До приймального покою доставлений хворий 48-ми років, що знайдений на вулиці взимку. Об’єктивно: сонливий, рівень свідомості - оглушення, температура - 31°C, шкіра блідоціанотична, холодна, з мармуровим відтінком в ділянках ліктьових та колінних суглобів, Ps- 54/хв., АТ-95/60 мм рт.ст., ЧД- 8/хв., поверхневе. Який найбільш імовірний діагноз? A 48-year-old patient found on the street in winter was brought to the waiting room. Objectively: sleepy, level of consciousness - stupor, temperature - 31°C, the skin is pale cyanotic, cold, with a marble shade in the areas of the elbow and knee joints, Ps - 54/min., BP - 95/60 mm Hg, BH - 8/min., superficial. What is the most likely diagnosis?

Переохолодження середнього ступеня Moderate hypothermia

Отруєння чадним газом Monoxide poisoning

Геморагічний інсульт Hemorrhagic stroke

Ішемічний інсульт Ischemic stroke

Гіпертонічний криз Hypertensive crisis

158 / 200
Пацієнт 19-ти років госпіталізований з приводу ЧМТ. Під час огляду у пацієнта розвинувся напад тонічних судом правої руки без втрати свідомості, який тривав 2 хвилини. На очному дні - прояви застою дисків зорових нервів. КТ головного мозку виявила субдуральну гематому в лівій тім’яній ділянці. Поясніть механізм розвитку епілептичного нападу: A 19-year-old patient was hospitalized for a TBI. During the examination, the patient developed a tonic convulsion attack of the right arm without loss of consciousness, which lasted 2 minutes. On the day of the eye - manifestations of stagnation of optic nerve discs. CT scan of the brain revealed a subdural hematoma in the left parietal area. Explain the mechanism of development of an epileptic attack:

Подразнення кори головного мозку гематомою Irritation of cerebral cortex with hematoma

Травматичне пошкодження (руйнування) головного мозку Traumatic damage (destruction) of the brain

Метаболiчнi порушення Metabolic disorders

Підвищення внутрішньочерепного тиску Increased intracranial pressure

Гемодинамічні порушення Hemodynamic disorders

159 / 200
У хлопчика 15-ти років після перенесеної щойно вітряної віспи з’явився різкий біль у лівій нозі. Нижня кінцівка збільшилась у об’ємі, підвищилась температура тіла до 37,8°C. На лівій нозі шкіра з внутрішньої поверхні червона, вздовж судин при пальпації відмічається різкий біль та ущільнення тканин. Обвід гомілки та стегна зліва більший на 2 см порівняно з аналогічними частинами тіла справа. Які потрібно зробити додаткові діагностичні інструментальні дослідження? A 15-year-old boy developed a sharp pain in his left leg after recently having chicken pox. The lower limb increased in size, the body temperature rose to 37 .8°C. On the left leg, the skin from the inner surface is red, along the vessels during palpation, sharp pain and compaction of tissues are noted. The circumference of the lower leg and thigh on the left is 2 cm larger compared to similar parts of the body on the right. What additional diagnostic instrumental studies should be performed?

Ультразвукова доплерографія Ultrasound dopplerography

Рентгенографія X-ray

Аускультація Auscultation

Ангіографія Angiography

Томографія Tomography

160 / 200
В приймальне відділення лікарні доставлено хворого з гострою затримкою сечі. Яке інструментальне обстеження найбільш інформативне? A patient with acute urinary retention was brought to the hospital admission department. Which instrumental examination is the most informative?

Цистоскопія Cystoscopy

Сцинтіграфія Scintigraphy

УЗД органів малого тазу та заочеревинного простору Ultrasound of pelvic organs and retroperitoneal space

Хромоцистоскопія Chromocystoscopy

Екскреторна урографія Excretory urography

161 / 200
У хворого зі стенозом пілоричного відділу шлунка з приводу виразкової хвороби останні 2 місяці спостерігаються безперервні блювання. Схуднув на 15 кг, зневоднений. Поступив у тяжкому стані, у приймальному відділенні з’явилися судоми. Яке обстеження необхідно провести для діагностики ускладнення? A patient with stenosis of the pyloric part of the stomach due to peptic ulcer disease has had continuous vomiting for the last 2 months. He has lost 15 kg, he is dehydrated. He was admitted in a serious condition, in the receiving department convulsions appeared. What examination should be performed to diagnose the complication?

УЗД органів черевної порожнини Ultrasound of abdominal organs

УЗД, визначення рівня кальцію в крові Ultrasound, determining the level of calcium in the blood

Комп’ютерна томографія Computed tomography

Рентгенографія шлунка Stomach X-ray

Визначення гематокріту Determination of hematocrit

162 / 200
З метою швидкого поповнення крововтрати хворому перелито 1000 мл одногрупної резуссумісної донорської крові. Наприкінці гемотрансфузії у хворого з’явилися неспокій, блідість шкірних покривів, тахікардія, судоми м’язів. Яка причина появи даної симптоматики? In order to quickly replenish blood loss, the patient was transfused with 1,000 ml of single-group Rh-compatible donor blood. At the end of the blood transfusion, the patient developed restlessness, pallor of the skin, tachycardia, muscle cramps . What is the reason for the appearance of this symptomatology?

Гемотрансфузійний шок Transfusion Shock

Пірогенна реакція Pyrogenic reaction

Цитратна інтоксикація Citrate intoxication

Тромбоемболія Thromboembolism

Анафілактична реакція Anaphylactic reaction

163 / 200
Пацієнт 25-ти років госпіталізований у відділення інтенсивної терапії через 30 хвилин після ДТП у стані важкого травматичного шоку. Пацієнту виконано лапароцентез з діагностичним перитонеальним лаважем. Отримано зворотній тік злегка геморагічного каламутного ексудату. В ексудаті виявлені волокна клітковини з різким неприємним запахом. Про що це свідчить? A 25-year-old patient was hospitalized in the intensive care unit 30 minutes after a road accident in a state of severe traumatic shock. The patient underwent a laparocentesis with diagnostic peritoneal lavage. The reflux was slightly hemorrhagic cloudy exudate. Fiber fibers with a sharp unpleasant smell were found in the exudate. What does this indicate?

Пошкодження сечового міхура Bladder damage

Пошкодження паренхіматозного органу Damage of a parenchymal organ

Травматичних уражень внутрішніх органів нема There are no traumatic lesions of internal organs

Пошкодження кишки Intestinal damage

Пошкодження підшлункової залози Pancreas damage

164 / 200
Дитина народилася від матері, яка хвора на цукровий діабет. Навколоплідні води були меконіальними. Дихання у дитини відсутнє, гіпотонія м’язів. Хлопчик поміщений під лампу променевого тепла, забезпечене правильне положення голови, проведено відсмоктування слизу з верхніх дихальних шляхів, дитина заінтубована. Які подальші дії реаніматолога? A child was born to a diabetic mother. The amniotic fluid was meconium. The child was not breathing, hypotonia of the muscles. The boy was placed under a radiant heat lamp, the correct position of the head is ensured, suction of mucus from the upper respiratory tract is performed, the child is intubated. What are the further actions of the intensivist?

Оксигенотерапія вільним потоком Free Flow Oxygen Therapy

ШВЛ через інтубаційну трубку Ventilation through an intubation tube

Непрямий масаж серця Indirect heart massage

Відсмоктування меконію трубкою, обсушування, шВл Aspiration of meconium with a tube, drying, shVl

Введення адреналіну ендотрахеально Introduction of epinephrine endotracheally

165 / 200
Хвору 20-ти років доставлено у тяжкому стані до стаціонару з ознаками отруєння парацетамолом. Який антидот необхідно ввести хворій? A 20-year-old patient was brought to the hospital in serious condition with signs of paracetamol poisoning. What antidote should be administered to the patient?

Ацетилцистеїн Acetylcysteine

Натрію тіосульфат Sodium thiosulfate

Атропіну сульфат Atropine sulfate

Налоксон Naloxone

Метиленовий синій Methylene blue

166 / 200
Дитина 11-ти років грала у футбол та під час падіння отримала травму. Встановлено попередній діагноз: закритий вивих правого передпліччя. Який обсяг допомоги повинен надати лікар на місці пригоди? An 11-year-old child was playing football and was injured during a fall. The preliminary diagnosis was established: closed dislocation of the right forearm. What amount of help should the doctor provide at the scene of the accident ?

Знеболювання, транспортна іммобілізація шиною Anesthesia, transport immobilization with a splint

Знеболювання, спроба вправлення вивиху Anesthesia, attempt to repair dislocation

Восьмиподібна пов’язка на правий ліктьовий суглоб Eight-shaped bandage on the right elbow joint

Знеболювання, транспортна іммобілізація косинкою Anesthesia, transport immobilization with a scarf

Косинкова пов’язка на праву верхню кінцівку Handkerchief bandage on the right upper limb

167 / 200
Чоловік 64-х років був притиснутий бампером автомобіля до стіни. Об’єктивно: відмічається патологічна рухливість в ділянці лонного зчленування, виражена болючість і припухлість над лоном, порушення сечопуску. Як слід транспортувати хворого? A 64-year-old man was pressed against the wall by a car bumper. Objectively: pathological mobility in the area of ​​the pubic joint, pronounced soreness and swelling above the pubic area, urination disorders are noted How should the patient be transported?

Лежачи на правому боці Lying on right side

Лежачи на спині, ноги зігнути в колінних і кульшових суглобах, стегна трохи розвести в боки Lying on your back, bend your legs at the knee and hip joints, spread your hips slightly to the sides

Лежачи на лівому боці Lying on the left side

Лежачи на спині з випрямленими нижніми кінцівками Lying on the back with straightened lower limbs

Лежачи на животі, підклавши під груди i голову подушку або одяг Lying on your stomach with a pillow or clothes under your chest and head

168 / 200
При аварії на ядерному реакторі відбувся викид в навколишнє середовище значної кількості радіоізотопів йоду. Яка добова доза йодиду калію показана для профілактики ураження щитоподібної залози? In the event of a nuclear reactor accident, a significant amount of radioisotopes of iodine was released into the environment. What daily dose of potassium iodide is indicated for the prevention of thyroid damage?

0,25 г 0,25 г

0,1 г 0,1 г

0,5 г 0,5 г

1 г 1 г

0,125 г 0,125 г

169 / 200
Роділля 36-ти років знаходиться в третьому періоді шостих термінових пологів. За відсутності ознак відділення плаценти через 10 хвилин розпочалася маткова кровотеча, яка перевищує фізіологічну і продовжується; втрата свідомості. Яка тактика лікаря? A 36-year-old woman in labor is in the third period of her sixth term delivery. In the absence of signs of placental separation, uterine bleeding began 10 minutes later, which exceeds physiological and continues; loss of consciousness What are the doctor's tactics?

Клемування за Генкелем-Тіканадзе Clamping according to Henkel-Tikanadze

Масаж матки на кулаці Massage of the uterus on the fist

Гемотрансфузія Hemotransfusion

Введення в вену метилергометрину Introduction of methylergometrine into a vein

Ручне відшарування плаценти та виділення посліду, лікування геморагічного шоку Manual detachment of the placenta and separation of litter, treatment of hemorrhagic shock

170 / 200
У хворого скарги на гарячку, озноб, різкий біль у правій пахвинній ділянці, запаморочення, блювання. Об’єктивно: обличчя і кон’юнктиви яскраво-червоні, язик сухий вкритий білим нальотом. В правій пахвинній ділянці різко болючий бубон, шкіра над ним червона, блискуча. З анамнезу: 3 дні тому повернувся у складі геологічної партії з Монголії. Про яке захворювання можна думати? The patient complains of fever, chills, sharp pain in the right groin area, dizziness, vomiting. Objectively: the face and conjunctivae are bright red, the tongue dry, covered with white plaque. In the right inguinal area, a sharply painful drum, the skin above it is red, shiny. From the anamnesis: returned 3 days ago as part of a geological party from Mongolia. What kind of disease can you think of?

Чума Чума

Туляремія Tularemia

Геморагічні гарячки Hemorrhagic fevers

Сибірка Anthrax

Лімфаденіт Lymphadenitis

171 / 200
Чоловік 49-ти років потрапив у ДТП. Об’єктивно: стан важкий, міжреберні проміжки згладжені, ліва половина грудної клітки відстає при диханні. Над лівим легеневим полем тимпаніт. Аускультативно: дихання зліва не проводиться, ЧДР- 32/хв. Інші пошкодження відсутні. Як транспортувати потерпілого? A 49-year-old man was involved in a road accident. Objectively: the condition is severe, the intercostal spaces are flattened, the left half of the chest lags behind during breathing. Tympanitis over the left lung field . Auscultation: no breathing from the left side, CHDR- 32/min. There are no other injuries. How to transport the victim?

Грудна клітина фіксується шинами Кра-мера до щита The chest is fixed with Kramer's tires to the shield

На правому боці лежачи Lying on the right side

В напівсидячому чи напівлежачому положенні In a semi-sitting or semi-lying position

В положенні Волковича на щиті In the position of Volkovich on the shield

На лівому боці лежачи Lying on the left side

172 / 200
У хворого 74-х років на 3 добу гострого інфаркту міокарда стан погіршився. Раптово знепритомнів. Пульс відсутній, дихання немає. На ЕКГ: часті нерегулярні хвилі, різні за формою та амплітудою. Після проведення реанімаційних заходів серцева діяльність була відновлена. Які препарати треба додати до терапії? On the 3rd day of an acute myocardial infarction, a 74-year-old patient's condition worsened. He suddenly fainted. There is no pulse, no breathing. On the ECG: frequent irregular waves, different form and amplitude. After resuscitation measures, cardiac activity was restored. What drugs should be added to the therapy?

Дексаметазон Dexamethasone

Атропін Atropine

Адреналін Adrenaline

Дигоксин Digoxin

Кордарон Кордарон

173 / 200
Хвора 32-х років зазнала тривалого впливу опромінення організму в малих дозах (0,1-0,3 Гр на добу) при сумарній дозі, що перевищує 0,7-1,0 Гр. Які наслідки даного опромінення? A 32-year-old patient was exposed to long-term exposure to low-dose radiation (0.1-0.3 Gy per day) with a total dose exceeding 0, 7-1.0 Gy. What are the consequences of this exposure?

Хронічна променева хвороба Chronic radiation sickness

Дана доза є летальною для людини This dose is lethal for humans

Гостра променева хвороба Acute radiation sickness

Гостра променева хвороба з переходом у хронічну форму Acute radiation sickness with transition to chronic form

Дана доза не є шкідливою для організму This dose is not harmful to the body

174 / 200
У хворої 45-ти років грип з важким перебігом ускладнився вірусно-бактеріальною двобічною пневмонією та розвитком гострого дистрес-синдрому. Сатурація кисню 70%. Хвора переведена до реанімаційного відділення. Що необхідно провести хворій? A 45-year-old patient had a severe flu complicated by viral-bacterial bilateral pneumonia and the development of acute distress syndrome. Oxygen saturation 70%. The patient was transferred to the intensive care unit What should be done to the patient?

Штучна вентиляція легень Artificial lung ventilation

Трахеостомія з аспірацією Tracheostomy with aspiration

Бронхоскопія з аспірацією Bronchoscopy with aspiration

Катетеризація підключичної вени Catheterization of the subclavian vein

Непрямий масаж серця Indirect heart massage

175 / 200
Постраждалий евакуйований з зони військових дій. Під час огляду: АТ- 110/90 мм рт.ст., Ps- 100/хв., індекс Альговера складає 1. Постраждалий дещо загальмований, але легко вступає в контакт, реагує на біль. Шкірні покрови бліді, дихання прискорене, втрата крові близько 800 мл. Який ступінь геморагічного шоку в хворого? The victim was evacuated from the war zone. During the examination: blood pressure - 110/90 mm Hg, Ps - 100/min, Algover's index is 1 . The victim is somewhat inhibited, but easily comes into contact, reacts to pain. The skin is pale, breathing is accelerated, blood loss is about 800 ml. What is the degree of hemorrhagic shock in the patient?

ІІ ІІ

І І

- -

ІІІ ІІІ

IV IV

176 / 200
Дитина 12-ти років постраждала під час ДТП. Мають місце численні переломи кінцівок, забій органів грудної клітки, ознаки травматичного шоку ІІІ ступеня. Який препарат необхідно використати для знеболювання під час транспортування в стаціонар? A 12-year-old child was injured in a road accident. There are numerous fractures of the limbs, bruised chest organs, signs of traumatic shock of the III degree. What drug should be used for pain relief during transportation to a hospital?

Тіопентал натрію Sodium thiopental

Пропофол Propofol

Кетамін Ketamine

Оксибутірат натрію Sodium oxybutyrate

Севофлюран Sevoflurane

177 / 200
У хворого 65-ти років виник 6іль в тазу, що іррадіює в промежину при сечовипусканні. У хворого протягом тривалого часу в зв’язку з аденомою передміхурової залози був введений уретральний катетер. Біль посилюється під час сечовипускання, яке стало частим, періодично спостерігається раптове 'перекриття'струменя сечі. Яке обстеження з метою диференційної діагностики слід виконати в першу чергу? A 65-year-old patient developed a cyst in the pelvis, radiating into the perineum during urination. The patient was injected for a long time due to prostate adenoma urethral catheter. The pain worsens during urination, which has become frequent, and sudden 'overlap' of the stream of urine is periodically observed. What examination for the purpose of differential diagnosis should be performed first of all?

УЗД органів малого тазу Ultrasound of pelvic organs

Рентгенографія органів малого тазу X-ray of pelvic organs

КТ органів малого тазу CT of pelvic organs

Лапароскопія Laparoscopy

Цистоскопія Cystoscopy

178 / 200
Жінка 64-х років з нестабільною стенокардією, під час ходи раптово впала. Об’єктивно: відсутність свідомості, пульсації на магістральних артеріях та тонів серця; вузькі зіниці та поверхневе дихання. З чого в даному випадку доцільно розпочати реанімаційні заходи? A 64-year-old woman with unstable angina suddenly fell while walking. Objectively: lack of consciousness, pulsation on the main arteries and heart sounds; narrow pupils and shallow breathing. In this case, why is it advisable to start resuscitation measures?

Внутрішньовенне введення адреналіну Intravenous administration of epinephrine

Інтубація та проведення ШВЛ Intubation and ventilation

Застосування дефібрилятора Defibrillator Application

Внутрішньовенне введення атропіну Intravenous administration of atropine

Удар кулаком по груднині A punch to the sternum

179 / 200
У хворого під час операції на шлунку було ушкоджено селезінкову артерію, в результаті чого виникла масивна кровотеча, яка призвела до зупинки кровообігу. Найдоцільніше проводити: The patient's splenic artery was damaged during a stomach operation, as a result of which massive bleeding occurred, which led to the cessation of blood circulation. It is most expedient to carry out:

Введення гідрокортизону Injection of hydrocortisone

Абдомінальна компресія Abdominal compression

Електрокардіостимуляція Electronic cardiostimulation

Прямий масаж серця Direct heart massage

Непрямий масаж серця Indirect heart massage

180 / 200
У постраждалого з важкою комбінованою травмою констатовано зупинку дихання, розширення зіниць, відсутність пульсу на магістральних судинах. Які негайні заходи необхідно вжити при асистолії? A victim with a severe combined injury has stopped breathing, dilated pupils, and lack of pulse on the main vessels. What immediate measures should be taken in case of asystole?

Введення β-адреноблокаторів Introduction of β-blockers

Інфузія поліглюкіну Polyglukin infusion

Внутрішньовенне введення натрію гідрокарбонату Intravenous administration of sodium bicarbonate

Штучне дихання, закритий масаж серця Artificial respiration, closed heart massage

Введення кортикостероїдів Introduction of corticosteroids

181 / 200
Хворий 65-ти років страждає на цукровий діабет І типу, раптом знепритомнів. Бригадою ШМД за допомогою глюкометра встановлено, що рівень глюкози крові становить 1,2 ммоль/л. При огляді хворий знаходиться в стані коми. Який препарат можливо застосувати окрім глюкози? A 65-year-old patient suffers from type 1 diabetes and suddenly fainted. Using a glucometer, the medical team determined that the blood glucose level was 1.2 mmol/l . During the examination, the patient is in a state of coma. What drug can be used besides glucose?

Тіопентал натрію Sodium thiopental

Естрадіол Estradiol

Глюкагон Glucagon

Адреналін Adrenaline

Мікседол Mixedol

182 / 200
Першовагітна 18-ти років прийшла на прийом. Строк вагітності 28 тижнів. Жінка соматично здорова. Вагітність перебігає без ускладнень. Під час огляду на кушетці жінка зблідла, шкіра стала вологою, з’явилися різка слабкість, нудота, знепритомніла. AT- 80/50 мм рт.ст., Ps- 88/хв., слабкого наповнення і напруження. Яка перша допомога в даній ситуації? An 18-year-old pregnant woman for the first time came for an appointment. The pregnancy is 28 weeks old. The woman is physically healthy. The pregnancy is going without complications. During the examination on the couch, the woman turned pale, the skin became wetness, sudden weakness, nausea, fainted. AT- 80/50 mm Hg, Ps- 88/min, weak filling and straining. What is the first aid in this situation?

Повернути жінку на лівий бік, потім її посадити Turn the woman to her left side, then sit her down

Ввести мезатон 1,0 мл п/ш Enter mesatone 1.0 ml f/w

Обкласти вагітну грілками, напоїти гарячим чаєм Cover the pregnant woman with heating pads, give her hot tea

Ввести 1,0 мл 0,2% розчину норадреналіну в 0,5 л 5% р-ну глюкози Inject 1.0 ml of 0.2% norepinephrine solution in 0.5 l of 5% glucose solution

Ввести розчин 5% глюкози 50,0 мл в/в Enter 5% glucose solution 50.0 ml IV

183 / 200
Чоловіка 42-х років доставили до лікарні після ДТП з відривом лівої гомілки. Об’єктивно: шкіра бліда, холодна на дотик, АТ- 40/0 мм рт.ст., Ps- 140/хв, ЧД- 32/хв. На лівій нижній кінцівці артеріальний джгут. Знеболювання не проводилось. Яка причина тяжкого стану чоловіка? A 42-year-old man was brought to the hospital after a road accident with a torn left leg. Objectively: the skin is pale, cold to the touch, blood pressure - 40/0 mm Hg .st., Ps- 140/min, ChD- 32/min. There is an arterial tourniquet on the left lower limb. Analgesia was not administered. What is the cause of the man's serious condition?

Гостра серцева недостатність Acute heart failure

Травматичний шок Traumatic shock

Тромбоемболія легеневої артерії Thromboembolism of the pulmonary artery

Жирова емболія Fat embolism

Масивна крововтрата Massive blood loss

184 / 200
Дівчині 17-ти років після масивної маткової кровотечі почата гемотрансфузія. Після введення близько 70-80 мл крові у хворої з’явилися інтенсивні болі у попереку; виділила сечу червоно-лакового кольору. Яка причина стану, що розвився у хворої? A 17-year-old girl started a hemotransfusion after massive uterine bleeding. After injecting about 70-80 ml of blood, the patient developed intense lower back pain; she passed red urine - lacquer color. What is the cause of the condition that developed in the patient?

Хворій перелито іншогрупну кров The patient was transfused with blood of a different group

Остеохондроз Osteochondrosis

Гострий гломерулонефрит Acute glomerulonephritis

Радикуліт Sciatica

Ниркова коліка Renal colic

185 / 200
При огляді дівчинка 7-ми років раптово знепритомніла. Шкіра та слизові блідоціанотичні. Екскурсій грудної клітини немає. Пульс на магістральних артеріях не визначається. Розпочато реанімаційні заходи. Яке дослідження необхідно використати для визначення серцевого ритму? During the examination, a 7-year-old girl suddenly fainted. The skin and mucous membranes are pale cyanotic. There are no chest excursions. The pulse on the main arteries is not determined. Resuscitation measures have been started. What a study should be used to determine the heart rate?

ЕКГ ЕКГ

УЗД УЗД

Ангіографія Angiography

ЕЕГ ЕЕГ

Пульсоксиметрія Pulse oximetry

186 / 200
У дитини 12-ти років з нервово-артритичним дiатезом після вживання в їжу значної кількості м’яса із кетчупом раптово виник гострий переймоподібний біль у животі та попереку, який іррадіює у статеві органи. При сечовипусканні отримано каламутну сечу із домішками крові. Найбільш імовірна причина гематурії? A 12-year-old child with neuro-arthritic diathesis, after eating a large amount of meat with ketchup, suddenly developed acute spasm-like pain in the abdomen and lower back, which radiates to the genitals. When urinating, cloudy urine with blood impurities was obtained. The most likely cause of hematuria?

Гострий гломерулонефрит Acute glomerulonephritis

Нефритичний синдром Jade Syndrome

Ниркова колька Renal colic

Геморагічний цистит Hemorrhagic cystitis

Травматичне ушкодження сечовивідних шляхів Traumatic injury of the urinary tract

187 / 200
Спостерігається значний викид радіонуклідів в атмосферу пошкодженим ядерний реактором. Очікувана доза опромінення всього тіла за перші 10 діб становить 50 мЗв. Яких захисних заходів треба вжити? There is a significant emission of radionuclides into the atmosphere by a damaged nuclear reactor. The expected dose of radiation to the whole body for the first 10 days is 50 mSv. What protective measures should be taken?

Ніяких, тому що дози х-опромінення не перевищують допустимі рівні None, because x-radiation doses do not exceed permissible levels

Обмежити вживання забруднених радіонуклідами харчових продуктів та питної води Limit consumption of food and drinking water contaminated with radionuclides

Провести йодну профілактику Conduct iodine prophylaxis

Використовувати укриття та засоби захисту органів дихання та шкірного покрову Use shelter and respiratory and skin protection

Евакуація населення із зон радіоактивного забруднення Evacuation of population from areas of radioactive contamination

188 / 200
Особи, котрі брали участь у ліквідації наслідків ядерної аварії, отримали дозу опромінення 20 мЗв. Якої медичної допомоги вони потребують? Persons who participated in the liquidation of the consequences of a nuclear accident received a radiation dose of 20 mSv. What medical assistance do they need?

Використовуються укриття та засоби захисту органів дихання та шкірного покриву Shelter and means of protection of respiratory organs and skin are used

Ніяку, тому що дози х-опромінення не перевищують допустимі рівні None, because x-radiation doses do not exceed permissible levels

Переселення або евакуація населення Relocation or evacuation of population

Потрібно провести йодну профілактику It is necessary to carry out iodine prophylaxis

Обмежити вживання забруднених радіонуклідами харчових продуктів та питної води Limit consumption of food and drinking water contaminated with radionuclides

189 / 200
Чоловік доставлений у медичну роту через 2 години після ядерного вибуху. Скаржиться на головний біль, болі у животі, м’язах, суглобах, озноб, нудоту, блювання, рідкі випорожнення. Усі симптоми з’явилися через 5-7 хвилин після опромінення. Індивідуальний дозиметр відсутній. При огляді загальмований, адинамічний. Шкіра та слизові оболонки гіперемовані, склери іктеричні, язик сухий. Ps- 100/хв., слабкого наповнення, визначаються поодинокі екстрасистоли, тони серця глухі, АТ-100/50 мм рт.ст., дихання везикулярне, живіт болісний під час пальпації, температура тіла 39,2°C. Який найбільш імовірний діагноз? A man was taken to the medical center 2 hours after the nuclear explosion. He complains of headache, pain in the stomach, muscles, joints, chills, nausea, vomiting, liquid stools. All symptoms appeared 5-7 minutes after exposure. There is no individual dosimeter. On examination, he is inhibited, adynamic. The skin and mucous membranes are hyperemic, the sclera is icteric, the tongue is dry. Ps- 100/min., weak filling, isolated extrasystoles, dull heart sounds, blood pressure 100/50 mmHg, vesicular breathing, abdomen painful during palpation, body temperature 39.2°C. What is the most likely diagnosis?

Гостра променева хвороба, кістково-мозкова форма, IV ступінь важкості Acute radiation sickness, bone-brain form, IV degree of severity

Гостра променева хвороба, кишкова форма, IV ступінь важкості Acute radiation sickness, intestinal form, IV degree of severity

Гостра променева хвороба, кишкова форма, II ступінь важкості Acute radiation sickness, intestinal form, II degree of severity

Гостра променева хвороба, кишкова форма, ІІІ ступінь важкості Acute radiation sickness, intestinal form, III degree of severity

Гостра променева хвороба, кістково-мозкова форма, III ступінь важкості Acute radiation sickness, bone-brain form, III degree of severity

190 / 200
Хвора 46-ти років після емоціонального навантаження раптово відчула сильний біль за грудниною. Об’єктивно: шкіра бліда, акроціаноз, кінцівки холодні. АТ-100/60 мм рт.ст. На ЕКГ: підйом сегменту ST над ізолінією у відведеннях aVL, V2-V6. Який найбільш імовірний діагноз? A 46-year-old patient suddenly felt severe pain behind the sternum after emotional stress. Objectively: pale skin, acrocyanosis, cold extremities. AT-100/60 mm Hg. On the ECG: elevation of the ST segment above the isoline in leads aVL, V2-V6. What is the most likely diagnosis?

Невралгія Neuralgia

Інфаркт міокарда Myocardial infarction

Синдром Дреслера Dresler Syndrome

Перикардит Pericarditis

ТЕЛА BODIES

191 / 200
У хворого 28-ми років субфебрильна температура, біль у животі, рідкі випорожнення з домішками слизу до 3-4 разів на добу. Хворіє протягом 3-х діб. Язик вологий, обкладений білим нальотом. Живіт м’який, болючий в лівій здухвинній ділянці, сигмоподібна кишка спазмована, болюча. Який метод дослідження буде найбільш інформативним для постановки діагнозу? A 28-year-old patient has low-grade fever, abdominal pain, loose stools with mucus up to 3-4 times a day. He has been ill for 3 days. The tongue is moist, coated with a white coating. The abdomen is soft, painful in the left pubic region, the sigmoid colon is spasmodic, painful. What research method will be the most informative for making a diagnosis?

Копроцитограма Coprocytogram

Копрокультура Coproculture

Загальний аналіз сечі General urinalysis

РНГА РНГА

Загальний аналіз крові General blood test

192 / 200
Хворий 20-ти років лікується з приводу гострого гепатиту мікст (В+D). На 8-й день захворювання у нього різко зросла жовтяниця, з’явилися дезорієнтація, психомоторне збудження, а через короткий час - втрата свідомості, патологічні рефлекси та мимовільне сечовиділення і дефекація, сумнівні менінгеальні знаки. Визначте прогноз стану хворого: A 20-year-old patient is being treated for acute mixed hepatitis (B+D). On the 8th day of the disease, his jaundice increased sharply, disorientation appeared , psychomotor excitement, and after a short time - loss of consciousness, pathological reflexes and involuntary urination and defecation, dubious meningeal signs. Determine the prognosis of the patient:

Одужання Recovery

Хронізація хвороби Chronicity of the disease

Несприятливий Unfavorable

Сприятливий Favorable

Сумнівний Doubtful

193 / 200
Хлопчик у віці 6 місяців хворіє на тяжку форму кашлюка: число нападів кашлю становить 25, напади тяжкі, тривалістю до 10 хвилин, мають 6-8 репризів, закінчуються блюванням. Один з нападів кашлю призвів до апное. Які невідкладні дії'? A 6-month-old boy has a severe form of whooping cough: the number of coughing attacks is 25, the attacks are severe, last up to 10 minutes, have 6-8 repetitions, end with vomiting . One of the coughing fits led to apnea. What are the immediate actions'?

Налагодити подачу кисню Set up oxygen supply

Седуксен в/м Seduxen v/m

Штучне дихання (в ручному режимі) Artificial respiration (in manual mode)

Лобелін в/м Lobelin v/m

Преднізолон в/м Prednisone IM

194 / 200
На фоні гіпертермії та сильного головного болю в дитини 5-ти років виникли зорові галюцинації жахливого змісту, марення, дезорієнтація. Шкіра гіперемована, з дрібною висипкою, рожевого кольору. Шкіра носогубного трикутника бліда, ознаки напруження м’язів потилиці, симптоми Керніга, Брудзинського, пожвавлення рефлексів без різниці між сторонами. Який діагноз можна припустити? Against the background of hyperthermia and a severe headache, a 5-year-old child developed visual hallucinations of terrible content, delirium, disorientation. The skin is hyperemic, with a small rash, pink in color. The skin of the nasolabial triangle is pale, there are signs of tension in the muscles of the back of the head, symptoms of Kernig, Brudzinsky, invigoration of reflexes without difference between the sides. What diagnosis can be assumed?

Деліріозний синдром при алергічному захворюванні Delirious syndrome in allergic disease

Деліріозний синдром при черепно-мозковій травмі Delirious syndrome in brain injury

Коревий менінгіт із деліріозним синдромом Measles meningitis with delirious syndrome

Деліріозний синдром при отруєнні їжею Delirious syndrome with food poisoning

Менінгококовий менінгіт з деліріозним синдромом Meningococcal meningitis with delirious syndrome

195 / 200
У хворого 29-ти лікар на шкірі передпліччя виявив різко болючий інфільтрат діаметром до 2 см, перифокальний набряк, лімфангоїт, лімфаденіт. У крові: лейк.- 13 • 1012/л, п- 20%. Три дні тому хворий повернувся з Індії. З хворим контактувало 3 особи. Тактика лікаря відносно контактних: In a 29-year-old patient, the doctor found a sharply painful infiltrate up to 2 cm in diameter on the skin of the forearm, perifocal edema, lymphangitis, lymphadenitis. In the blood: leuk.- 13 • 1012/l, p- 20%. The patient returned from India three days ago. 3 people came in contact with the patient. The doctor's tactics in relation to contacts:

Госпіталізувати в провізорний госпіталь Hospitalize in temporary hospital

Направити в госпіталь особливо небезпечних інфекцій Send particularly dangerous infections to the hospital

За контактними не спостерігати Do not monitor contacts

Медичне спостереження в амбулаторних умовах Medical observation in outpatient settings

Госпіталізувати в обсерваційне відділення Hospitalize in the observation department

196 / 200
У пацієнта 40-ка років через 3 дні лікування інфаркту міокарда, раптово пів години тому виникли болі у правій нозі та моторна афазія, втрата глибокої чутливості, рухової активності правої ноги нижче коліна. Визначте засіб невідкладної терапії: In a 40-year-old patient, after 3 days of treatment for a myocardial infarction, pains in the right leg and motor aphasia, loss of deep sensitivity, motor activity of the right legs below the knee Determine the means of emergency therapy:

Введення папаверину Introduction of papaverine

Введення дезагрегантів та інфузійна терапія Introduction of antiplatelet agents and infusion therapy

Введення гепарину 10 000 ОД внутрішньовенно Introduction of 10,000 units of heparin intravenously

Введення наркотичних анальгетиків Introduction of narcotic analgesics

Введення тромболітиків внутрішньовенно Introduction of thrombolytics intravenously

197 / 200
Медсестра Центру профілактики і боротьби зі СНІДом під час маніпуляції травмувалася голкою інфузійної системи. Які препарати можуть бути призначені для постконтактної профілактики? The nurse of the AIDS Prevention and Control Center was injured by the needle of the infusion system during manipulation. What drugs can be prescribed for post-contact prophylaxis?

Антиретровірусні препарати Antiretroviral drugs

Антибактеріальні препарати Antibacterial drugs

Така профілактика не проводиться Such prevention is not performed

Імуноглобуліни Immunoglobulins

Препарати інтерферону Interferon drugs

198 / 200
На борту літака, який прибув з центральноафриканської країни перебуває хворий з підозрою на карантинне захворювання. Які протиепідемічні заходи є першочерговими з боку санітарно-епідеміологічної служби аеропорту? There is a patient suspected of having a quarantine disease on board the plane that arrived from a Central African country. What anti-epidemic measures are the priority of the airport's sanitary-epidemiological service?

Лабораторне обстеження усіх прибулих літаком на попередньо діагностовану карантинну інфекцію Laboratory examination of all arrivals by plane for pre-diagnosed quarantine infection

Пасажирам та екіпажу розпочати неспецифічну профілактику імовірного захворювання Passengers and crew to start non-specific prophylaxis of suspected disease

Ізоляція екіпажу та пасажирів літака на 23 дні Isolation of the crew and passengers of the plane for 23 days

Відведення літака на спеціальний санітарний майданчик та ізоляція хворого в мельцерівський бокс Removal of the plane to a special sanitary area and isolation of the patient in the Meltzer box

Залишити усіх прибулих у літаку до встановлення діагнозу Keep all arrivals on the plane until diagnosis

199 / 200
У хворого з підозрою на поширену дифтерію мигдаликів у приймальному відділенні раптово погіршився стан: виникли лихоманка, прогресуюча різка слабкість, похолодання кінцівок на фоні гіпертермії, мармуровість шкіри, тахікардія, зниження артеріального тиску. Які дії лікаря приймального відділення? The condition of a patient suspected of widespread diphtheria of the tonsils in the receiving department suddenly worsened: fever, progressive sharp weakness, cooling of the extremities against the background of hyperthermia, marbling of the skin, tachycardia, decrease in blood pressure. What are the actions of the doctor in the reception department?

Інтубація Intubation

Транспортування хворого у ВРІТ інфекційної лікарні Transportation of a patient to the ICU of an infectious disease hospital

Негайне переведення хворого у ВРІТ Immediate transfer of the patient to the ICU

Ін’єкція антибіотиків широкого спектру дії Injection of broad-spectrum antibiotics

Госпіталізація хворого в інфекційне відділення Hospitalization of the patient in the infectious department

200 / 200
Хворий чоловік 20-ти років скаржиться на високу гарячку, біль у литкових м’язах, жовтяницю. Об’єктивно: збільшена печінка, позитивний симптом Пастернацького. У крові підвищений вміст білірубіну, азотистих метаболітів. Який найбільш імовірний діагноз? A sick man in his 20s complains of high fever, pain in the calf muscles, jaundice. Objectively: an enlarged liver, positive Pasternacki's symptom. In the blood increased content of bilirubin, nitrogenous metabolites. What is the most likely diagnosis?

Вірусний гепатит Viral hepatitis

Черевний тиф Typhoid

Лептоспіроз Leptospirosis

Геморагічна лихоманка Hemorrhagic fever

Грип Flu